You are on page 1of 40

1

ECONOMICS
Higher 2
(Syllabus 9732)
Suggested Answer Outlines and
Examiners Comments for the
Year 6 Preliminary Examinations 2012

Year 6 Preliminary_9732 [Turn over]


RI 2012
2

Case Study Question 1: Transport Issues in Singapore

Mark Scheme to Case Study 1


a i. Using Figure 1, compare the ridership between the different modes of transport [2]
between 2005 and 2010.

[1]: Identify similarity in ridership trend over time for bus and MRT or difference in trend between bus 1
and taxi or MRT and taxi

Examples that can secure 1 mark:


There is a rising trend for average daily ridership for bus and MRT but a fall in average daily
ridership for taxi over the period.

There is a rising trend for average daily ridership for both bus and MRT

There is rising trend for average daily ridership for bus but a fall in average daily ridership for
taxi over the period.

MRT/LRT ridership shows the greatest increase.


[1]
[1]: Identify relative position in ridership between bus and taxi

Examples that can secure 1 mark:


Average daily ridership is highest for travel by bus across all the other modes of transport while
it is the lowest for taxi ridership over the period.

Average daily ridership is highest for travel by bus

Average daily ridership is lowest for taxi ridership

Examiners comments: Most candidates lost marks because they failed to compare the relative position
in ridership between the different modes of transport. There was also an inaccurate explanation that all
modes of transport increased.

ii. With the aid of a diagram, account for the trend in train ridership from 2002 to 2010. [4]

[1]: Increase in demand (population size increase or increase in price of substitutes (cost of owing 1
private vehicles or any other demand-side factor from the case material)

[1]: Increase in supply (expansion of rail network eg. More MRT lines) 1

[2]: Use DD-SS diagram and shift the demand curve to the right and the supply curve to the right and 2
show the increase in train ridership on the quantity-axis
Increase in train ridership

[0 out of 4]: Diagram without explanation

Examiners comments: Some scripts lost credit as they failed to recognize the key words With the aid of
a diagram. In addition, it was disappointing that some candidates still only gave a one-sided answer
(i.e. only demand or supply side factors but not both). Candidates are also reminded that it is good
practice for them to provide a brief explanation of how the information in the case evidence will lead to
an increase in demand or supply for rail services. There is also no need to discuss the price adjustment
process in this question as the question is only asking for the trend in train ridership (quantity traded).

Year 6 Preliminary_9732 [Turn over]


RI 2012
3

b Using Figure 2, to what extent is the pricing policy practiced by SMRT a type of price [6]
discrimination?

1m: Definition
1m: Identify type of PD
2m: Well-explained thesis
2m: Well-explained anti-thesis
Maximum 5m if there is no conclusion

Introduction
[1]: Define PD - Definition must be accurate and complete to secure one mark 1
- Charging different prices for the same good for reasons not associated with cost differences.

[1]: Identify the type of PD


rd nd
- 3 degree PD or 2 degree PD (depends on how candidates argue) 1

[2]: Thesis:
rd
Explain why the pricing policy is can be classified as 3 degree PD by appealing to the conditions for PD
- Max 2m: Differences in PED: From Table 2 there is a higher train fare for adults but 2
concessionary rates for both elderly and students. Adults are charged a higher price given that
their demand is relatively price inelastic. This is likely because the fares would constitute a
smaller proportion of their income.
- Max 1m: If explanation points only to No resale and/or firm must have monopoly power
and/or markets must be segmented (without further elaboration)

[2]: Anti-thesis
However, difference in price between standard ticket fares and EZ link Adult fares may not be due to
price discrimination. There may be actual cost differences due to the extra costs of hiring workers to 2
sort out the coins and notes from the standard ticket machines, while via EZ-link, it is done
electronically. [Tham Yan Ping. 12S06L]

OR

However, due to increased train service frequency during peak timings (after 7.45am) vs off peak
timings (before 7.45am) during weekdays, there could be higher (average variable/marginal) cost
incurred by the rail operators during the peak timings. Hence, this example is not necessarily a case of
price discrimination. [Peak load pricing argument]

Award max 1m for the following reasons accounting for price differences include the following:
- Children take up less space than adults
- Government subsidises the fares of elderly and children

Conclusion (Max 5m out of 6 if there is no conclusion):


Hence the difference in price between standard ticket fares and EZ link Adult fares that is practiced by
SMRT is most likely a combination of price discrimination and price differences. It involves price
discrimination with regards to the higher prices adults are charged compared to the elderly and children
based on differences in price elasticities of demand between the different groups of customers.
However, it is not an example of pure price discrimination because of the differences that can arise from
cost differences.

Note:
Peak-load pricing can be considered second degree price discrimination. This is where people are
charged more at times of peak demand and less at off-peak times. The reason for the higher prices
charged at peak times has partly to do with elasticity of demand (and in this sense, therefore, is price
discrimination: i.e. charging different prices because of different demand elasticities in different parts of
the market). Demand is less price elastic at peak times. For example, many commuters have little option
but to pay higher rail fares at peak times, that is, it is a necessity to them. Alternatively, PTO offers more
price sensitive commuters lower prices to travel during off-peak times in order to reduce congestion
Year 6 Preliminary_9732 [Turn over]
RI 2012
4

during peak periods. Since the firm is not able to sort out or identify who the more price sensitive
customers are and who are less price sensitive in advance, this can be considered second degree price
discrimination.

However, peak-load pricing may not be purely due to price discrimination. Frequently the higher charges
also have to do with higher marginal costs incurred at peak times. With various fixed factors (such as
plant and equipment), marginal costs are likely to rise as output expands to meet higher demand. This
could be due to diminishing returns to the variable factors/overutilisation of fixed factors. Or it could be
due to having to use additional equipment with higher operating costs.

Examiners comments: A number of candidates lost a mark as they failed to provide a precise and
complete definition of price discrimination. Many also lost a mark due to a lack of a conclusion. It is
essential to have a conclusion for questions that involve a discussion. In addition, candidates lost marks
as they also failed to provide an anti-thesis. For questions with the command words To what extent, a
balanced discussion is required. Diagrams are not required for this question given the demands of the
question and paper, and the mark allocation (max 6m).

C The wages of low skilled workers like bus drivers have remained stagnant for a long time.
Discuss whether the Singapore government should consider implementing a minimum [8]
wage policy to raise the wages of low skilled workers in Singapore.

Introduction:
Clarify what is minimum wage policy
- Define minimum wage policy: An effective minimum wage rate is one that is legally set above
the free market wage rate (can illustrate it with a diagram)
Thesis:
Explain why the Singapore government should consider implementing a minimum wage rate
- Problem: Stagnant wages of low skilled workers. Implement the minimum wage policy to help
improve their standard of living and reduce income inequality and hence reduce inequity and
also to achieve other social goals
nd
o Given the problem of stagnant wages of low skilled wages (Extract 2, 2 paragraph),
implementing a minimum wage can help them to maintain a minimum standard of living,
especially given the rising cost of living in Singapore
rd
o Extract 2, 3 paragraph: There is a limit to productivity increases for some low-skilled
jobs. For example, there is a limit to the maximum number of trips a bus driver can take
without compromising on his own safety, the safety of his passengers and other road
users. If productivity has to rise first before wage rates rise, the low skilled workers
nd
whose wages have remained stagnant for a long time (Extract 2, 2 paragraph) may
find themselves unable to cope with the rising cost of living.
o Reduce income inequality between wages of low and high skilled workers in Singapore,
in view that Singapores gini coefficient is rising and the highest among all the
developed countries improve income equity reduce social tension and foster
greater social cohesion
o Reduce the stigma associated with such jobs which is one reason why Singaporeans
shun these jobs and employers have to resort to hiring foreign workers which in turn
may result in social problems like integration problems between Singaporeans and
foreigners
o Reduce the stigma associated with such jobs and maintain the dignity of those who
work in these jobs. These workers do not have to depend on welfare handouts.
- In the long run, efficiency can improve if firms are forced to mechanise or find ways to raise the
productivity of low-skilled workers
o Force employers to mechanise to raise the productivity of low-skilled workers so as to
reduce cost of production. As productivity of low-skilled workers rise, cost-push inflation
can be dampened in the long run.

Anti-thesis:
Explain why the Singapore government might not want to implement this policy:
- Distorts the free market Rise in real wage unemployment:
Year 6 Preliminary_9732 [Turn over]
RI 2012
5
nd
o Extract 2, 2 paragraph: More low skilled Singaporeans may find themselves out of a
job Qs of labour exceeds Qd for labour at the minimum wage rate (which is
implemented above the free market equilibrium) SOL for the unemployed will worsen
- Increase in cost-push inflation
nd
o Extract 2, 2 paragraph: Higher wages without increases in productivity push up
business costs, affect Singapores competitiveness, and cause higher inflation
increase in unit cost of production upward shift of the AS curve increase in cost
push inflation accompanied by a slower economic growth rising cost of living affects
SOL adversely, especially for those who are unemployed or on fixed wage incomes
- Slow down economic growth
nd
o Extract 2, 2 paragraph: push up business costs, affect Singapores competitiveness,
and cause higher inflation. This may hold back many companies eager to expand but
cannot find enough workers
Investments fall slower economic growth slower improvement in SOL
Singapores exports become less price competitive slower economic growth
slower improvement in SOL
- Difficult to remove once implemented

Conclusion/Evaluation:
- Provide an overall stand and explain the overall stand (explanation can be in the body)
o In the short run, minimum wage law will lead to trade-offs between objectives. However,
if firms mechanise or find other ways to improve the productivity of low-skilled workers,
reduction in income inequality can be accompanied by higher efficiency, lower inflation
and higher economic growth (that is, these goals can be compatible in the long run).
- Suggest an alternative policy measure that the Singapore government can consider in raising
the wage rates for low skilled workers and explain why it might be a better measure than
minimum wage law.
o E.g. Increase productivity of low skilled workers helps to increase wage rates for low
skilled workers and reduces the probability of incurring cost-push inflation in Singapore

Other possible evaluation:


In the SR, an appropriate one-off move to help improve real wages of low skilled workers and in turn
SOL but should be coupled with LT policies of raising productivity of worker such that increased
productivity levels relative to the higher wages can translate to lower cost of production and increased
output over time and to help cope with displaced workers arising from higher wages to re-enter the
workforce.
Or given that min-wage policy to help certain group within the economy may trade off goals of
employment levels / worsen unemployment (in particular, structural unemployment) and price stability
reduce SOL, the gov. should not implement this policy. Rather, government should seek towards raising
the productivity of these low skilled workers to ensure that there is higher returns to their wages in the
LR and coupled with a ST policies of increased vouchers / subsidies/ benefit scheme eg. medical
coverage? to help raise the purchasing power of low skilled workers.

L1 [1-3]: A one-sided answer on minimum wage law AND no economic framework AND no application
specific to Singapore AND no case evidence

L2 [4-5]: Lapses in coverage

L3 [6-8]: A well-balanced analysis on whether the Singapore government should consider implementing
the minimum wage law with case evidence, economic framework and/or application specific to
Singapore. Given mark allocation, two well-explained point on each side can secure full 8 marks.

If no conclusion, max 7/8

Examiners comments: This question was generally well answered. Most candidates are aware of the
need to define key economic concepts found in the question. In this case, they defined and explain the
concept of a minimum wage policy using the demand/supply diagram. They went on to analyse the
potential benefits of implementing such a policy and the potential costs it may have on the economy.
Year 6 Preliminary_9732 [Turn over]
RI 2012
6

However, candidates generally lost marks when they failed to explain what a minimum wage policy is. In
addition, they also lost marks when they failed to use the case evidence to support their arguments.
There was also a tendency for candidates to lift case evidence from the extract and not explain the
statements with the use of economic theory. Such scripts tended to score poorly.

A few comments were disturbing. For instance, a few candidates wrote that the minimum wage law will
cause low skilled workers to become complacent and less hardworking. This might be true if the
minimum wage law is accompanied by a law that makes it difficult to fire workers once they are hired.
However, Singapore does not have a law that makes it difficult to fire workers. Given the real wage
unemployment generated by the minimum wage law, it is more likely that those who are retained in
employment will have greater incentive to work harder lest they lose their jobs. In addition, given the
initial wages of the low skilled, the substitution effect of a rise in wage (opportunity cost of leisure is
higher encouraging more work effort) is likely to outweigh the income effect (only need to work less to
maintain the same standard of living).

d The rail industry can be said to be a natural monopoly. [10]


Discuss the desirability of the various forms of government intervention in such an
industry.

Introduction: Define natural monopoly.


A natural monopoly occurs in an industry where LRAC and LRMC falls over a wide range of output
levels such that there may be room only for one supplier to fully exploit all of the internal economies of
scale, reach the minimum efficient scale and therefore achieve productive efficiency.

Explain why the rail industry is a natural monopoly (but not necessary to do so.)
From Extract 3, due to the high fixed / overhead costs such as capital outlay for rail infrastructure, eg.
MRT stations, tunnels and the maintenance of the rail system, the rail industry is considered a natural
monopoly with significant internal economies (technical) of scale. This means that one large operator
can supply public transport services at lower costs than two or more smaller operators which in this case
apply to the distinctly different rail networks owed by the 2 private operators, SMRT and SBS. The LRAC
and LRMC curves of the natural monopoly slopes downwards relative to market demand.

There are various forms of government intervention in a natural monopoly.

1) Privatisation + Regulation: The government can choose to privatize the natural monopoly and
regulate it by appointing a regulator to ensure that the good remains affordable to all its citizens on
the grounds of equity, especially since rail travel is a necessity. The government can implement the
following policies:

i) Price regulation: MC pricing or AC pricing policy or Cap-price regulation

Extract 3: Fare regulation involves the regulator, in the case, the Public Transport Council to oversee
fares that the public transport operator (PTO) could charge. In theory, the government can regulate the
industry by requiring the PTO to set prices at marginal cost or at average cost. Referring to figure 1, with
marginal cost pricing, social optimum is achieved at Qr because consumers marginal benefit from the
last unit sold is equal to the marginal cost of producing that last unit. By setting prices at average cost,
however, the monopolist will be able to breakeven (lowest effective price), but the output level at Qf will
be less than social optimum at Qr.

Year 6 Preliminary_9732 [Turn over]


RI 2012
7

Figure 1: Price Regulation-MC and AC pricing

LAC

From Extract 3: the government has established a fare regulator- PTC to oversee the review of fares so
as to prevent the private operators from abusing their market power to set excessively high fares but
rather after adjusting for both wage increases and inflation.

Desirability:
Increases allocative efficient outcome, reduces price, equity: Compared to unregulated profit
maximising price, both average cost and marginal cost pricing reduce price and increase output,
increasing consumers surplus and social welfare.

Undesirable/Limitations :
Losses incurred by the PTO: With MC pricing, the natural monopolist may have to face losses
unless government subsidies are given to the producer or 2-tier pricing is practiced.
The demand and cost curves can only be estimated and the regulated firm may withhold or distort
information. Example, a utility firm may overstate its costs so it can charge more.
rd
In reality, according to Extract 3, 3 paragraph, fares are regulated based on an adjustment formula
which takes into account factors such as the Consumer Price Index (CPI) and average national wage
increases. This protects commuters by capping fare increases, so that PTOs are not able to simply pass
their cost increases to the public.
th
ii) Extract 3, 4 paragraph: PTOs are likely to face stiffer fines for lapses in service in the
future as the government reviews the current $1 million maximum fine.
Advantages and disadvantages of legislation.

iii) Impose a lump-sum tax on the supernormal profits of the firms and redistribute the tax
revenue to the lower income families on the grounds of equity so that lower income families
can spend the direct subsidy that they receive on rail travel. Any reasonable limitation.

2) Nationalise the rail industry (Government operators to run the rail services)

The government often takes over natural monopolies to prevent monopoly pricing, especially if the
goods are essential or of strategic national interest, in this case, public transport. Nationalisation can be
defined in several ways, for example, it could mean the government taking over transport planning,
ownership of infrastructure, or service provision. In this article, nationalisation is defined specifically as
government provision of public transport services.

Desirability:
Increase allocative efficiency increased consumer welfare through lower fares and better service
/ quality of buses. From Extract 3: Government-run public transport operators which are not profit-
driven would be less likely to abuse their market power, and would place greater priority on public
service objectives, such as affordable fares and service enhancement.

Increase productive efficiency reduced duplication less wastage of resources.. Nationalization

Year 6 Preliminary_9732 [Turn over]


RI 2012
8

also increases productive efficiency by eliminating the overlapping activities of competing suppliers.
In industries supplying services such as electricity, water and transportation, duplication is wasteful
and inefficient. From Extract 2: If competently run, nationalisation could reduce the costs associated
with the duplication of functions and roles

Promote Equity: Government can cross-subsidise unprofitable routes deemed unprofitable for
private operators to ply. Although operating these unprofitable routes necessitates cross subsidies
from other routes, it will result in a new level of connectivity to the more remote areas. This
improvement will not only provide connectivity for residents living on the outskirts but also enhance
the value of their surrounding real estate.

Undesirable/Limitations:
Productive Inefficient: Extract 3, 5 paragraph: Nationalisation will cost even more for the
th

Government to run the services by themselves. The lack of a profit maximizing motive provides little
incentive for government operators to be productive efficient. Besides, the bureaucratic nature of
management in nationalization creates over-large and over-bureaucratic organizations which,
therefore, suffer from diseconomies of scale. Hence, government operators may fail to operate
efficiently and incur high cost of production. They would also face greater political pressure to
provide unprofitable services.

Inequity: Nationalized industries are in effect public monopolies. It can be argued that a state
monopoly could be just as disadvantageous to the consumer as a private one. This is because there
is no higher authority to protect the consumers interests. The consumer therefore has to tolerate the
lack of choice and perhaps even high prices with little hope of redress, although there are normally
consumer consultative bodies.
th
3) (Extract 3, 5 paragraph): Letting PTOs bid for licenses on a shorter term, and the threat of
replacement after expiry of the licence, can incentivize the incumbent PTOs to maintain good
performance. promotes competition in bidding for licenses to operate in the industry. This will
reduce X-inefficiency. Although PTOs have a natural monopoly on their individual lines after the
license is given, there is a threat of replacement if they are inefficient. This incentivizes them to
reduce wastage, benefitting society. However, one drawback is that shorter licence terms can
prevent operators from making long-term investments in better signaling devices and ensuring good
maintenance of tracks beyond their term. [Tham Yan Ping. 12S06L]

Conclusion:
There are different approaches to providing rail services, each with its merits and shortcomings. The role
of the government is to aim to develop integrated and accessible rail transport systems with reasonable
quality of services and at affordable fares. These ideally have to be fiscally sustainable, and provided by
efficient and financially viable operators. However, given the potential tradeoffs of the above objectives
of efficiency and equity, the government has to strike a balance between them.

In conclusion, it is desirable for government intervention in Singapore because of the high degree of
market failure in the natural monopoly, which is the failure to achieve efficient allocation of resources
and other social goals like equity. Given that rail services are deemed to be a basic necessity, the
government has to protect equity in Singapore. However, nationalization is not desirable as it can scare
off foreign investors which Singapore is highly dependent on for growth. A combination of regulation and
introducing competition in the form of bidding for licences on a shorter term to operate in the industry
should be used, while bearing in mind the possibility of government failure due to imperfect information.
[Adapted from Tham Yan Ping]

L1 [1-3]: A one-sided answer on one form of intervention AND no economic framework AND no case
evidence AND lack understanding of what is a natural monopoly.
Explains what a natural monopoly is and/or the aims of intervening in such an industry but yet to
address the forms of intervention.
L2 [4-5]: Lapses in coverage and rigour
Max 5: Pros and cons of only 1 policy (some links to case evidence) that is well-explained
L3 [6-8]: A well-balanced analysis on the different forms of intervention in a natural monopoly and the
Year 6 Preliminary_9732 [Turn over]
RI 2012
9

rail industry. Includes rationale, advantages and disadvantages of 2 distinctly different policies from the
case study. Analysis done within a strong economic framework (rigour of analysis and scope of
coverage). One advantage and one disadvantage of each policy is sufficient to score full L3=8. Should
consider affordability/equity, efficiency and quality of service in the answer.

E[1-2]:
Max 2: Make a stand on which is a better policy for the rail industry in Singapore for example,
privatization or nationalisation and explain why. Or suggest alternative policies and explain why
Max 1: Make a stand on which is a better policy but without reference to the rail industry
Max 1: An attempt at a conclusion that addresses the question meaningfully

No conclusion: Max 1 mark for evaluation

Examiners comments: This question was very poorly done. A significant majority of the candidates failed
to use the concept natural monopoly in their answers, even though it was clearly stated in the question.
For those who attempted to explain the concept, there were flaws in their diagrammatic illustration, such
as rising LRAC and LRMC curves relative to market demand which is clearly not a trait associated with a
natural monopoly. The correct diagrammatic illustration for a natural monopoly should be one where the
LRAC and LRMC curves fall relative to market demand. A number even went on to discuss about the
public transport sector being a duopoly, which was not what the question wanted.

Many candidates were also unaware of the definition of a natural monopoly. Many also misinterpreted
the question and discussed instead the desirability of government intervention in such an industry,
instead of the desirability of the various forms of government intervention in such an industry.

It was good to see many candidates using case materials in their answer. However, they are reminded
that there is a need to provide an economic framework in their answers. They must not provide a
journalistic response. In addition, they must not lift the materials from the text without explaining what
the case evidence means. There is also a need for candidates to provide a balance discussion in their
answers.

Case Study Question 2: Government Policies in the United Kingdom (UK)

a. i) Explain the meaning of public debt. [2]

Public debt is the total amount of financial liabilities (1m) held by the government (1m).

-1m for showing understanding that debt is a stock concept (e.g. sum of deficits, total amount, accumulation
of deficits etc.).
-1m for showing understanding that the liability is held by the government due to the government spending
beyond its revenue. Do not accept phrasing that still uses the word 'public' (e.g. held by public sector).

ii) Using the data in Table 2, explain the evidence that shows an increase in UKs real debt in
absolute terms from 2006 to 2008. [4]

Tables 1 shows that real GDP growth has been largely positive. This means that real GDP has increased
from 2006 to 2008 (1m). At the same time, the debt to GDP ratio has also increased (1m). Since real
absolute debt = (debt/GDP)*(real GDP), the increase in both debt/GDP and real GDP means real absolute
debt must have increased (2m).

-1m for making correct inference that real GDP has increased from real GDP growth rates
-1m for making correct reference to debt to GDP ratio
-2m for showing formula/reasoning and conclusion

Year 6 Preliminary_9732 [Turn over]


RI 2012
10

Examiners comments: i) A large number of candidates did not know that the term public sector does not
refer to the public. Other candidates are confused between the government budget position and balance of
payments position. The budget relates to governments revenue (taxes) and expenditure. The balance of
payments relates to inflow and outflow of money from the whole economy. In addition, only very few
candidates recognized that debt is a stock concept while deficit is a flow concept.
ii) It is very disappointing to see many students use the CPI data in the answer when real GDP has been
given. A large number of candidates did a summation of growth rates to obtain the total change over a period.
Please note that this method is an approximation and will only be accurate if the time period is short, thus it
should be avoided if possible. Additionally, doing a year-on-year comparison is a waste of time since the
question asks for the period 2006 to 2008. To see if debt has increased overall, it is sufficient to compare the
debt in the final period with that in the beginning period. Lastly, students should develop the good habit of
providing the formula with which they did their computations. This increases the clarity of their answers.

b) To what extent does UKs trade balance explain the external value of the pound in the period
shown in Figure 3? [6]

Introduction: Briefly mention how exchange rates are determined in the forex market.
Demand for pound Demand for UK exports or financial assets.
Supply of pound Demand for foreign imports or financial assets.
Under a floating exchange rate system, the equilibrium value of the pound is determined at the
intersection of demand and supply.

Thesis: The general trend in the external value of the pound is consistent with the general trend in
the trade balance.
Figure 2 shows the trade deficit generally worsening over the period downward pressure on the
exchange rate, ceteris paribus depreciation of the pound in Figure 3.

Anti-thesis: The trade balance does not fully explain the extent of short-term changes in the external
value of the pound (Elaborate on any 1 of the periods shown below).

From 2008 to 2009


UKs trade balance deficit worsened slightly but there was a sharp depreciation of the pound.
The sharp depreciation was probably due to QE as pounds were converted to foreign currencies to
buy foreign assets (Ex. 6)

From 2009 to 2010


UKs trade balance deficit worsened quite significantly compared to the year before but there was a
very slight depreciation of the pound.
This was probably due to increased confidence in pound denominated assets in comparison to Euro
denominated assets which were more affected by the debt problem in Greece (Ex.5)

Judgement/Synthesis:
Trade balance only partially explains movements in the exchange rates. Changes in the financial account will
play a significant role especially in view of the globalisation of capital.

3m for thesis
3m for anti-thesis
Both thesis and anti-thesis should make use of case material and link the points to the analytical framework
of demand and supply of currency for full range of marks.

Examiners comments: In explaining how BOT affects value of GBP, many simply stated that a BOT deficit
means value of M is greater than value of X and therefore GBP depreciates. The link between the demand
for GBP and that of British goods by foreigners is absent in many scripts. It is NOT good enough to state the
relationship. The candidate must demonstrate ability to explain and illustrate to show the grasp of higher
order skills. Many candidates also misread the question and tried to explain whether the depreciation could
explain the change in the BOT instead of whether BOT could explain the depreciation which was what the
question required. Candidates must read the question carefully and follow the demands of the question.

Year 6 Preliminary_9732 [Turn over]


RI 2012
11

Additionally, inflation should not have been provided as an alternative factor that explained the depreciation
as inflation works through affecting the BOT. Furthermore, a good number of candidates also confused
money supply and supply of money in the foreign exchange market. The former is the supply of money within
an economy. The latter is the supply of domestic currency in the foreign exchange market. The two may be
related: flows between both markets can be either way but they are NOT identical flows. Lastly, some
candidates only pointed out the incongruence of the BOT and NEER data (e.g. the former improved but the
latter depreciated) in their anti-thesis but failed to give an alternative factor that could have explained the
change in NEER. Candidates should learn to provide alternative factors that could explain the relationship
when the data is incongruent.

c) i) Why does the Bank of England set an inflation target? [2]

BoE sets an inflation target to influence expectations (1m) to achieve a stable and low rate of inflation. (1m)

-1m for recognizing the mechanism of influencing expectations to maintain confidence and risk taking in
the economy. Creating certainty, inspiring confidence, signaling, etc are all acceptable.
-1m for the explicit aim of inflation targeting (ideas associated with improved monetary policy efficiency due
to the focus and commitment on keeping inflation at a certain rate once a target is announced so its easier
to obtain inflation outcomes closer to target levels are acceptable).

ii) Why is unanticipated inflation often considered more of a problem than anticipated inflation? [6]

Inflation is a persistent rise in the general price level. Anticipated inflation happens when economic agents
(households, firms and government) are able to make accurate predictions of inflation while unanticipated
inflation is seen as inflation that is volatile from year to year such that it becomes difficult for economic
agents to correctly predict the rate of inflation in the near future. Unanticipated inflation occurs when
economic agents (i.e. people, businesses and governments) make errors in their inflation forecasts.
Actual inflation may end up well below, or significantly above expectations

There are costs associated with both unanticipated and anticipated inflation e.g. worsened income
distribution.

Unanticipated inflation is often considered more of a problem as it is associated with costs that are generally
more severe.

The costs of anticipated inflation are largely only menu and shoe-leather costs where menus have to be
updated more frequently due to the changing prices, and more trips having to be made to withdraw more
cash for transactions.

The costs of unanticipated inflation include uncertainty, confused price signals, which could lead to
misallocation of resources when producers mistake an increase in the price of their product as an increase in
the relative price of their product and wrongly allocate more resources to its production and unproductive
speculative activities undertaken by economic agents in their attempt to try to protect themselves.

The costs of anticipated inflation are less severe than unanticipated inflation as
modern technology has largely reduced such costs (e.g. card payments reduced shoe-leather costs).
Economic agents can take steps to protect themselves. Indexation can be used so that incomes and
taxes retain their real value. For example, trade unions may exercise their collective bargaining power to
negotiate with employers for increases in money wages so as to protect the real wages of union members.
Households may also be able to switch savings into deposit accounts offering a higher nominal rate of
interest or into other financial assets such as housing or equities where capital gains over a period of time
might outstrip general price inflation. In this way, people can help to protect the real value of their financial
wealth. Companies can adjust prices and lenders can adjust interest rates. Businesses may also seek to
hedge against future price movements by transacting in forward markets. For example, most of the major
airlines buy their aviation fuel several months in advance in the forward market, partly as a protection against
fluctuations in world oil prices.

Year 6 Preliminary_9732 [Turn over]


RI 2012
12

-1m for showing understanding of difference in unanticipated and anticipated inflation


-3m for explaining the costs of inflation (one well-explained cost is sufficient)
-2m for explanation of factors that causes the costs of anticipated inflation to be lower.
Explanations must be linked back to examples used to score full 2m (e.g. if example of cost of anticipated
inflation is worsened income inequality, then explanation of how taking it into account can lead to lower costs
would be wages can be inflation-indexed to prevent decrease in real-income of fixed wage earners).

Examiners comments:

c. i) Most candidates did make use of Extract 4 and attempted to link inflation targeting to an attempt by the
monetary authorities of a country to commit to an announced figure so that expectations could be managed
and no wage-price spiral will occur if the target is credible.

ii) Majority of the candidates did not address question posed properly and answered an imaginary question
of this form: Explain the benefits of anticipated inflation and the costs of unanticipated inflation. In addition,
very few used the example from context of trade union asking for increase in wages to compensate for
higher inflation when it is anticipated. Many simply regurgitated all they have memorized from lecture notes.
Lastly, policies to mitigate costs of inflation and policies to reduce inflation are irrelevant here. Governments
use contractionary policies to fight inflation that is unacceptably high (that is, inflation was unanticipated and
beyond targeted rate if there is inflation targeting in the first place), not fight anticipated inflation. Thus,
contractionary policies to bring unacceptable inflation to acceptable levels have no relevance to the question
posed.

d. With reference to both the case evidence and your own relevant knowledge, discuss the likelihood
of UK slipping into a recession in 2012. [8]

1. Whats recession?

2. Thesis: UK unlikely to slip into a recession in 2012 as


With inflation targeting and setting expectations: certainty and maintenance of confidence and risk
taking so C and I boosted and AD rises
QE: successfully increase C. I and also (X-M): AD rises. BoE may also have further rounds of QE.
Exchange rates have been decreasing (Fig 3). This stimulates external demand and leads to growth.
Trade balance: slowly showing improvement over time and if the trend continues, will be in
equilibrium
Positive GDP growth seen from 2010 to 2011.

3. Thesis: UK likely to slip into a recession after 2011 as


Austerity drive: AD falls as G falls, T rises, C falls and I also falls.
VAT rise: cost of production rises and this causes inflation to rise. A shift of the AS to the left can
derail any rise in AD that increases GDP.
External environment is not bright: continued Euro crisis and USs economic recovery is also not
very strong.
Impact of QE might well be negated by other events in the world economy, for example falling
demand and confidence and a return to recession in the euro area or the depressive effects of high
oil and gas prices on real incomes and spending.

4. Conclusion:

Possible evaluation:
1. Very difficult to estimate and predict based on existing data due to changing conditions
2. Ultimately depends on extent of each policys effects. Political considerations could curtail the extent
to which austerity measures are imposed (the coalition government is highly unpopular at the moment. The
Tories are behind Labour in the polls and Lib Dems are all but non-existent).
3. Expectations is the key with a general gloom over EU, it is unlikely that stimulus measures will be
very effective (especially since UK mainly exports to rest of EU).

Year 6 Preliminary_9732 [Turn over]


RI 2012
13

4. QEs impact may be limited as it mainly increases wealth of shareholders/stock owners. These tend
to be the rich with lower MPC.

L1 Smattering of points with no/little economics analysis/ framework 1m-


Journalistic writing mainly case information 3m
One sided, i.e. discuss either why recession or why no inflation
No application to context/case
L2 Evidence of economic analysis 4m
Attempt at a balanced answer (thesis & anti-thesis) 6m
Some application to context / case
Insufficient depth or insufficient scope of analysis (lacks the discussion of external
factors)
L3 Well-balanced answer with rigour 7m
Clear use of economic framework 8m
Apt reference to case evidence
E1 Unexplained assessment made e.g. merely stating of synthesis 1m
E2 Evaluative assessment supported by economic analysis eg. well-justified synthesis 2m

Examiners comments: Some candidates could only describe and extrapolate the given economic
indicators provided in Table 2 and they made little reference to the use of government policies in the Extract.
Such candidates scored poorly as such simplistic extrapolation of data trends are insufficient when there are
clear factors in the case material that would cause AD to increase or decrease, depending on the implicit
assumptions and situations when the policies are carried out. In addition, many candidates used the
Marshall-Lerner condition without stating what it was (sum of elasticities of X and M greater than one). Lastly,
QE is carried out by Central Banks effectively printing money. The government does not incur any extra debt
as a result of QE. Some candidates also mistakenly classified QE as a supply side policy. Such carelessness
is to be avoided. Lastly, candidates must remember that it is insufficient to state an outcome: depth must be
seen by illustrating and analyzing how the outcome is reached. Thus, the use of an analytical framework to
help in achieving depth is very crucial to demonstrating higher order skills.

Year 6 Preliminary_9732 [Turn over]


RI 2012
14

Essay Section

1 Technological advancements such as in information technology and the London 2012 Olympics have
had major impacts on the demand and supply of Olympic-related products and services, for example
travel services, media services and food and beverages.

Assess how these developments might affect the relevant markets in Singapore. [25]

Introduction:
Introduce the benchmark that is usually used when analyzing changes in market conditions for any particular
market: sales revenue or consumer expenditure (price x quantity)
Introduce the various product markets and the possible submarkets that will be assessed. Relevant markets
will be those mentioned in the question directly as well as inter-related markets. If the particular market
happens to employ a key resource (oil; skilled labour) in its production, then that factor market could be
analyzed as well. Some examples can include:
1) Travel Services
a. Air Travel
b. Hotels in Singapore
2) Media Services
a. Pay TV
3) Food and beverage
a. Health drinks/Junk Food industries
b. Food outlets/restaurants in Singapore
Diagrams to illustrate the various effects on sales revenue / consumer expenditure arising from changes in
DD and SS conditions should be included and explained
Introduce the various elasticity concepts that will be used in the discussion:
1) Price elasticity of demand (to understand the impact on price and quantity when supply shifts)
2) Price elasticity of supply (to understand the impact on price and quantity when demand shifts)
3) Cross-price elasticity of demand (to understand the impact on inter-related markets)

Body
Thesis: Markets involved are affected positively (i.e. sales revenue increase)
I) Travel Services (Air Tickets from Singapore to UK)
Technological advancements
o Reduced costs of air travel (larger aircraft, better / more fuel efficient planes)
o Reduced ticketing costs
These would see an increase in supply of air travel; SS curve shifts to the right
o Ceteris paribus, holding demand constant and at the same time assuming demand for air travel is
price inelastic for Singaporeans travelling to London (few close substitutes for transport to London)
o A fall in equilibrium price leads to a less than proportionate increase in quantity. This will cause the
fall in revenue from the fall in price to be less than the gain in revenue from the increase in
quantity transacted
o Effect is a likely fall in total revenue for the air travel market
Increase in demand for air travel due to travellers wanting to travel to London, especially during the
period of the Olympic games (change in tastes and preferences); DD curve shifts to the right
o Holding supply constant and given that supply is price inelastic (limited by airlines capacity)
o A rise in equilibrium price will lead to less than proportionate increase in quantity.
o Effect is rise in total revenue for air travel market

Overall effect: sales revenue increase


o Although both DD and SS rise, it is likely that DD rises more than SS increases (no significant
advancements in air travel; reduction in costs of air travel is likely to be insignificant since the main
costs of air travel will be fuel charges) Diagram to illustrate the net effect after the various shifts of
DD and SS curves.

II) Media Services (pay TV)


Technological advancements (cheaper information and communications technology)
o Reduced costs of production for the pay TV provider (cheaper broadband / information and
Year 6 Preliminary_9732 [Turn over]
RI 2012
15

communications technology). Leads to increase in SS of media services; SS curve shifts to the right
o Ceteris paribus, holding DD constant and at the same time demand for pay TV (Olympics channels)
is price elastic (large variety of TV programs available)
o There will be a fall in equilibrium price and a more than proportionate increase in quantity for media
services
o Total revenue for the pay TV provider increases since the fall in revenue due to the fall in price is
less than the gain in revenue from the increase in quantity transacted.
OR
o Assuming demand for pay TV (Olympics channels) is price inelastic (it is a unique product)
o Total revenue for the pay TV provider is expected to fall since the fall in revenue from the lowered
price is greater than the gain in revenue from the increase in quantity
Increase in demand is expected for pay TV due to consumers wanting to watch Olympic related
programs (change in tastes and preferences)
o Holding supply constant and assuming supply is price elastic (once provided, it is easy to increase
the amount of pay TV to other subscribers)
o Effect is a rise in total revenue for pay TV providers

Overall effect: sales revenue expected to increase in the pay TV market (SS rise and DD rises)
Using cross-elasticity of demand: HD TV sets, as a complement to TV programs, may see an increase in
demand as consumers change their existing TV sets or purchase additional TV sets so that they are able
to better view the various Olympic events (sales revenue for the TV market will rise)

III) Food and Beverage (Examples analyzed are not exhaustive)


Health food
Technological advancements
o Reduced costs of production (better production processes). Leads to increase in SS of food
products; SS curve shifts to the right
o Assuming no change in demand conditions and demand for health food is price elastic (many close
substitutes availability of a variety of other food products apart from playing sports)
o There will be a fall in price of health food and a more than proportionate rise in quantity sold.
o Health food suppliers will experience an increase in total revenue since the fall in revenue from the
fall in price is less than the gain in revenue from the increase in quantity sold
Increase in demand may be experienced due to consumers wanting to consume more healthy food
(change in tastes and preferences)
o Holding supply constant and assuming supply to be price elastic (easier to increase the production of
many sports products)
o Effect is a rise in total revenue for health food suppliers

Overall effect: sales revenue increase (SS rises slower than DD increases)
Using cross-elasticity of demand: Health food are complements with sports equipment, hence an
increase in demand for health food can also cause an increase in demand for sports equipment
OR
Restaurants/Dining outlets with live screenings of the Olympics
Technological advancements
o Reduced costs of production (cheaper to install TV sets in dining outlets / cheaper TV subscriptions).
Leads to increase in SS of dining outlets with TV broadcasts of the Olympics
o Holding demand for restaurant dining constant and assuming its demand is price elastic (many close
substitutes many restaurants with similar facilities)
o There will be a fall in equilibrium price and a more than proportionate increase in quantity
o Total revenue is expected to increase as the loss in revenue from the fall in price is less than the
gain in revenue from the increase in quantity
Increase in demand for restaurant dining with live screening of the Olympics can be expected as there
may be a rise in consumers wanting to catch the Olympics programs while dining out (change in tastes
and preferences)
o Holding supply constant and given a price elastic supply (depends on the capacity of the restaurant)
o Effect is a rise in total revenue for these restaurant dining outlets

Year 6 Preliminary_9732 [Turn over]


RI 2012
16

Overall effect: sales revenue may or may not increase (depends on the rise in SS relative to the
increase in DD)

Anti-thesis: Markets involved are affected negatively (i.e. sales decrease)


I) Travel Services (Air Tickets from Singapore to other countries / hotels in Singapore)
Technological advancements
o Reduced costs of air travel (larger aircraft, better / more fuel efficient planes)
o Reduced ticketing costs
o These would see an increase in supply of air travel; SS curve shifts to the right
o Ceteris paribus, holding demand constant and assuming demand for air travel is price inelastic
o Net effect is a fall in total revenue for air travel companies as the fall in revenue from the fall in
price to be less than the gain in revenue from the increase in quantity transacted
Using cross-elasticity of demand: air travel by Singaporeans to other countries will fall. The fall in
demand is due to travellers wanting to travel to London, especially during the period of the Olympic
Games (substitute to air travel to London)
o Holding supply constant and supply is price inelastic (limited by airlines capacity)
Net effect is fall in total revenue earned from travel to other destinations

Overall effect: sales revenue fall (SS rise and DD falls)


o Likely that DD falls more than SS increases (no significant advancements in air travel; reduction in
costs of air travel is likely to be insignificant since the main costs of air travel will be fuel charges)
OR
Travel Services (Packaged Tours to UK/London)
Technological advancements in IT
o Reduced costs of purchasing air tickets / booking of hotels online
o Market for free-and-easy travel will experience a large increase in supply (costs are significantly
reduced as it is now easier to source for cheaper deals on the internet)
o Hence, supply of free and easy travel services increases. Holding demand constant, the price of
free-and-easy travel in the market will fall
Using cross-elasticity of demand: Free and easy travel is a substitute with tour packages
o Hence with a fall in price of free-and-easy travel, demand for tour packages to UK will fall
o The extent of the fall in demand and hence impact on TR will depend on value of its CED
o Analyze when they are close substitutes with each other (because they both serve the same
functions) or not close substitutes (travel agencies may offer other services like booking of tickets for
special concerts related to the Olympics events)
However, the Olympics Games as a whole increases the demand for travel to UK
o Demand will increase in the market for package tours to UK
Overall effect: Effect on sales revenue will depend on whether the increase in demand due to the
Olympics Games is greater than the fall in demand due to the fall in price in the free-and-easy
travel market)
o Consumer expenditure may fall (or rise correspondingly)
Note: One can also analyze that demand for travel on the whole may fall during the Olympics period as
consumers may shelve their holiday plans to stay home to catch the various Olympics events since the
Games happen only once every four years. CED concept may be used in the analysis.

II) Media Services (pay TV)


Technological advancements
o Cheaper for consumers to do online live streaming of the Olympics (in fact, most of it is free)
o There is a significant fall in price of online TV services
Using cross-elasticity of demand: online live streaming is a very close substitute to pay TV (same
service, but more flexible, one can even view the Olympics on ones computer/phone without a TV set)
o As a result, because online TV services is much cheaper and more convenient, demand for pay TV
will fall
Overall effect: it is likely that the fall in demand for pay TV is greater than the increase in its
demand (as discussed originally under the thesis) especially since the fall in price of the online
TV market is significant. We may expect consumer expenditure for pay TV to fall.

Year 6 Preliminary_9732 [Turn over]


RI 2012
17

III) Food and Beverage (junk food)


Technological advancements
o Reduced costs of production (better production processes)
o Demand for health junk food is price elastic (many close substitutes)
o An increase in supply together with a price elastic demand will see a fall in price and a more than
proportionate increase in quantity.
o Total revenue is likely to increase since the loss in revenue from the fall in price is less than the
gain in revenue from the increase in quantity.
Using cross-elasticity of demand: Can expect to see a fall in demand for junk food due to consumers
wanting to consume more healthy food (substitutes with health food)
o Given that supply of junk food is price elastic (easy to reduce the production of many sports
products), a fall in demand will bring about a fall in price and a more than proportionate fall in
quantity sold
o Net effect is fall in total revenue
Overall effect: sales revenue decrease (SS rises and DD falls)
o Fall in demand likely to be greater than the increase in supply (major costs of production is the costs
of raw materials used)
Note: candidates may also wish to highlight how some of these junk food producers (e.g. Coca-cola,
MacDonalds) may use the Olympics to aggressively advertise their products, and so increase the
demand for their products instead

OR
Restaurants/Dining outlets without live screenings of the Olympics
Technological advancements
o Unlikely to have an impact on the cost of production of restaurants since there are no changes to the
cost of raw materials and labour
Using cross-elasticity of demand: Dining outlets with no live screening may see a fall in its demand due
to consumers wanting to watch the Olympics (change in tastes and preferences) / (more consumers may
choose to dine in the restaurants with live screenings - substitutes to dining outlets with live screening or
stay home rather than eat out)
o assuming supply is price elastic (depends on the capacity of the restaurant), a fall in demand will
lead to a fall in price and a more than proportionate fall in quantity
o Net effect is fall in total revenue for these dining outlets
Overall effect: sales revenue decrease (SS rise insignificantly and DD falls)

Conclusion
Judgement as to which market will be affected the most by the developments.

Note: The answers for this question are not exhaustive. The above just provides a guide on the analysis
expected of the students.
Knowledge, Application, Understanding and Analysis
L1 Descriptive answer which contains little if any economics 19
Substantial/ lots of glaring conceptual errors throughout
Listing of points with little/no economic analysis
L2 Lapses in explanation: poor use of diagrams/use of basic demand and supply concepts 10 14
Max 12: Lack of Balance in discussion
Max 10: Failure to use any elasticity concepts;
L3 A balanced and rigorous examination of how the markets involved are affected by the 15 21
developments
Scope of coverage: 3 markets (covers goods and services) are addressed, the effects
of technological advancement and the impact of the Olympics is discussed
Rigour: Makes use of economic concepts (e.g. relevant elasticity concepts) with clear
and well-illustrated diagrams to explain the potential impacts on the demand curves.
Balanced: Discussion of how markets may be positively/negatively affected by the
developments
Max 18 marks: 2 markets well addressed (using 3 elasticity concepts)

Year 6 Preliminary_9732 [Turn over]


RI 2012
18

Evaluation
E1 For an evaluation not supported by analysis. 12
E2 For an evaluative assessment based on economic analysis which addresses the 34
question

Examiners Comments:
This question assesses the candidates ability to apply the various demand and supply and elasticity
concepts to various product markets.

Candidates are reminded that in order to score well for this type of question, they must display balance,
scope and rigour in their answer. As such, there are a few important areas they need to bear in mind:
- The markets given in the stimulus are general. There is a need to be specific about the markets in order
for students to have a meaningful discussion. Knowledge about possible sub markets within the
particular market discussed and the characteristics of some of the products in the particular market
would be very useful if one does not want to be vague in the discussion. For eg, many candidates could
focus on travel and media services but were clueless about what could fall under food and beverages.
As a result, they tend to leave that market out of their discussion or at most land up describing
MacDonalds marketing strategy of using the Olympics commemorative glasses to promote its value
meals.
- Use of relevant economic concepts in an answer is important. For eg. in analyzing changes in demand
and supply conditions within a particular market and its impact on TR or consumer expenditure, PED /
PES concepts will be required; CED will be useful when discussing inter-related markets. YED is not
useful in this question as there is no expected change in income and the time period is too short for
income changes to take place.
- The requirement of ceteris paribus assumption when applying elasticity concepts. The impact of a
change in supply on P and Q is analyzed using PED. Demand conditions must be held constant. If the
ceteris paribus assumption is not adhered to, the analysis becomes invalid.
- There is a need to see the link between the events and the identified product markets in the given
stimulus. Many candidates tend to forget about technological advancements and focused on the London
Olympics instead. Only a handful could identify technological advancements relevant to media (online
streaming, broadband) and food production.
- Analysis of the different markets will require different sets of diagrams. It is incorrect to group all the
product markets into 1 diagram to analyze.

Last but not least, question 1 is a broad question. The candidate has to be very clear in his/her structure of
the answer. Given the many possibilities of analysis, it can become quite complex and within 45 minutes,
breadth and depth can be sacrificed if one is unclear. The answers, most of the time, will land up superficial
and simplistic.

2 Market dominance in Singapore is frowned upon but has become increasingly necessary to compete
in a globalised world.

a. Explain possible reasons for the prevalence of oligopolistic markets in your country. [10]

b. Discuss whether large firms are beneficial to society in todays globalised world. [15]

Examiners Comments:

This question was generally well done. It was heartening to see that most students were able to identify the
key issues in the question, and provide an answer which reflected a good use of Economics as well as
suitable examples. Also, many students were able to provide a thoughtful evaluation to the issues in part b.
There were only a small minority which did not have sufficient time to complete the question. We will also like
to stress that as this question falls under Section A (Microeconomics), we expect students to be using
microeconomic concepts in their answers. An answer which discusses solely macroeconomic issues will not
earn much credit. Students are also reminded that they MUST define the key economic terms (internal
economies of scale, productive efficiency, allocative efficiency) and not just state them in their answers.
Year 6 Preliminary_9732 [Turn over]
RI 2012
19

Part (a) was a good discriminant between strong and weak students. Most students were able to provide
reasons and examples to account for the prevalence of oligopolies, but there were some gaps in their
explanations and so failed to attain the highest range of marks. For example, a number of students
discussed at length the IEOS that are present in the industry, but they failed to relate it to either the growth of
the firm (to earn more profits) or to how the large IEOS enables firms to practice predatory pricing and so
prevents new firms from joining the industry. Similarly, many students just stated that Singapore has a small
population which can only support few firms in the industry without explaining how the presence of more
firms will lead to firms having insufficient demand to obtain normal profits and survive. There were also a
number of students who highlighted that advertising will cause demand to be more price inelastic and so
makes it harder for entrants to compete, without explaining that the reason why entrants will find it harder to
compete is because they also need to do large scale advertising and so incur higher start-up costs.

There were still a number of strong scripts which were able to identify 3 distinct reasons for the prevalence of
oligopolistic markets in their country and explain them clearly with the aid of appropriate examples. A small
minority was only able to identify the characteristics of oligopolies and hence did not do well.

Some conceptual misunderstandings that were seen in the answers include:


The Minimum Efficient Scale (MES) is the lowest point on the LRAC. This is incorrect. The MES
refers to the lowest OUTPUT level at which all IEOS are exhausted.
The industry has high start-up costs and so the MES for the industry is large. This does not make
sense. MES is large only when there are substantial IEOS to be reaped, but the amount of IEOS
present in the industry does not depend on the high start-up costs. Students are expected to
elaborate and provide examples of IEOS that can be reaped in the industry in justifying why MES is
large for the industry.
The industry has high IEOS as the large fixed costs can spread over a larger output. This is wrong
as fixed costs are a short run concept while IEOS is a long run concept. A better way to express is
to write that the high start-up costs can be spread over a larger output.
Because of the high IEOS in the industry, large firms will then be productive efficient and operate
near the MES and so enjoy cost savings. This makes it desirable for there to be a few large firms in
the economy and that accounts for the prevalence of oligopolies. This is not true. Desirable DOES
NOT EQUAL to prevalence. There are many things in life which are desirable but not prevalent.
Part (b) was generally well done. Most students provided a balanced discussion of the economic benefits
and costs of having large firms in society. Many also recognised the global context of the question, and
addressed it. The conclusions provided were also well thought and covered a range of issues. However,
many scripts failed to provide any examples in their answers and so did not attain the highest level of marks.
It is important to have examples in your answers and not just provide a theoretical answer. A point to
highlight: you do not need examples throughout all your answers. The use of suitable examples at the right
parts (e.g. when discussing R&D or price setting ability) will be sufficient.

There were some scripts that deviated from the question and discussed the costs and benefits of
globalisation on large firms. These scripts inevitably did not score well. An important point to highlight is that
dumping, outsourcing, off-shoring and cross-country technology transfers are all issues associated with
globalisation, but not issues associated with large firms.

A number of students had an incomplete development of their explanations. Most students failed to
recognise that market power leads to a downwards sloping demand curve for large firms, and hence the
profit optimizing output level will occur where P > MC. Many students also did not explain why P=MC means
allocative efficiency. A number of students also did not use a diagram to explain allocative inefficiency when
it is essential. It is also important to highlight that the ability of large firms to do R&D is because they earn
LONG RUN supernormal profits due to high Barriers to Entry (and not due to market power). This is crucial
as all market structures are able to earn supernormal profits in the short run (even perfect competition). Also,
R&D which will lead to new and improved production techniques DOES NOT improve productive efficiency.
Productive efficiency refers to producing on the LRAC for a given state of technology. The reduction in costs
of production due to improvements in production techniques is part of dynamic efficiency.

Lastly, students are reminded that they are expected provide economic explanations for their answers. Some
students were able to mention that large firms may exploit workers by paying them low wages. This is true,
but you need to explain it using a framework (Monopsony power). The failure to do so will only lead to partial
Year 6 Preliminary_9732 [Turn over]
RI 2012
20

credit.

Possible mark scheme


(a) Explain possible reasons for the prevalence of oligopolistic markets in your country. [10]
Possible reasons (pick any 3 to explain, with examples)

Size of market in Spore is relatively small and is not able to support many firms. For example, the
public transportation system comprises only 2 rail companies and 2 bus companies and a handful of
taxi companies. Any new entrant will not be able to have a large enough demand to sustain normal
profits.
Advancement of technology which reduces production costs over large output, i.e. internal
economics of scale are often reaped only over large outputs, especially technical economies of
scale. Therefore firms will increase scale of production to reduce average costs of production and
therefore increase profits.
Globalisation leading to greater competition in the international market which leads to merger and
acquisition (M&A). Foreign firms may buy up domestic firms to gain greater foothold in the regional
market, eg Heineken buying up Asia Pacific Brewery (APB) and this reduces the no of breweries in
Singapore leading to increase in market share and fewer firms.
High startup costs in industries which restricts the number of firms possible in the industries as few
firms are capable of affording the capital needed to join the industry.
Artificial barriers to entry in the form of patents (and so other firms are prevented from producing the
good) or advertising/R&D (entrants into the industry must also conduct large scale advertising/R&D
in order for them to compete)
Internal Economies of Scale as a barrier to entry. The lower unit costs of production due to high
IEOS allows incumbents to charge lower prices for their goods. Entrants, with a higher unit cost of
production due to their small scale of production are unable to charge a low price and compete with
the incumbents. If they were to charge a low price, they will make subnormal profits and leave the
industry (predatory pricing).
Government policies:
- Number of firms is restricted by the need for government licenses to operate. This is usually
for essential services, eg the banking sector. There are only a couple of local banks which
can offer a full range of consumer banking services and a small no of foreign banks which
can operate in Singapore.
- Oligopoly is prevalent over monopoly because of anti-trust laws to prevent monopoly
formation. E.g., in telecommunications/internet industry which was dominated by Singtel for
many years, government issued license to other companies like, Starhub and M1 and
intervened when there was attempt by Singtel to monopolise in the provision of
telecommunications OR any proposed merger of large dominant firms requires governmentt
approval to ensure no formation of monopoly, e.g. when OUB merged with UOB and POSB
merged with DBS.
- Government subsidies to encourage small and medium enterprises to use greater
technology and so grow larger in size leading to the formation of oligopolies

Level Descriptor Marks


1 Listing of reasons with minimum explanation. 1-4
2 At least 2 well explained reasons using economic theory/ framework, with some attempt 5-6
at use of examples.
3 At least 3 well explained reasons and well elaborated with good use of examples in the 7-10
Singapore context. Strong use of economic theory/framework.

(b) Discuss whether large firms are beneficial to society in todays globalized world. [15]

Thesis: Benefits of large firms


- Greater productive efficiency (from societys point of view) due to the ability of large firms to
exploit fully all available IEOS (especially with globalisation, firms can now cater to a larger
Year 6 Preliminary_9732 [Turn over]
RI 2012
21

market). This leads to lower prices for consumers if the cost savings are passed on.
- Greater ability to compete in the international market due to the lower long run average costs
from IEOS. The cost savings if passed on will reduce the price of our exports, which will
increase external demand (exports increase) and so economic growth and employment will
increase. This is particularly critical to Spore which is very dependent on external market
because of its very small domestic market.
- Ability to do R&D because of long run supernormal profits. This leads to better products or
better production techniques (dynamic efficiency).
- Avoids duplication of resources. This is true especially in the utilities sector where we do not
need 2 sets of gridlines/pipes, and instead 1 large firm providing 1 set of gridline/pipes will help
to reduce wastage of resources.

Anti-thesis: Disadvantages of large firms


- Greater allocative inefficiency. Large firms have more market power and so a relatively price
inelastic demand. A profit maximizing firm will product output at which MR=MC, and so for a firm
with market power, this output will occur where P >MC. Since P = MC represents allocative
efficiency (value that society places on the good is equal to the opportunity cost of producing it),
a divergence between price and marginal cost will mean there is an underproduction of the good
and so large firms are allocative inefficient. (Diagram to illustrate)
- Less consumer choices. Consumers only have few firms to choose from, unlike in monopolistic
competition or perfect competition. In the case of monopoly, there is no choice.
- Inequity. The long run supernormal profits leads to a transfer of surplus away from the
consumers (poor) to the producers (rich). This will worsen the inequity caused by globalisation.
- Productive inefficiency (firms point of view). Large firms may be complacent and so X-inefficient
due to their long run supernormal profits. They may not see the need to drive down costs of
production and this will represent a wastage resources in society.
- Large firms may not be in the interest of society especially in industries where variety/personal
services are paramount to consumers, e.g. hair salons and fine-dining where personal attention
and details are critical. In these industries, small firms (e.g. Monopolistic competitive firms) are
better.

Evaluation: Need to balance micro and macro aims for a small open economy like Singapore.

The microeconomic objective of ensuring economic efficiency and protection of consumers welfare, though
critical, must be weighed against the macro objective of expanding economic growth into the international
market. One possible solution could be to open the domestic market to foreign competitors as we encourage
domestic firms to grow in size. This will ensure keen competition and therefore maintain consumers welfare
in terms of lower price, quality and more variety of goods and services. Also need to have social
programmes to address the widening income gap problem.

OR

Using the theory of contestable markets, the threat of entry by foreign competitors in a globalised world will
cause domestic large firms to be price competitive and so pass on cost savings to society. They will also
likely be more productive efficient (from firms point of view) and they may not abuse their market power that
much, thus operating closer to the allocative efficient outcome. In addition, the limitations of large firms can
be addressed using government policies such as price regulations (MC/AC pricing) to limit the abuse of
market power or lump sum taxes to address the issue of inequity. Furthermore, being a small and open
economy, large firms are essential in helping Singapore achieve its macroeconomic goals of greater
economic growth, low unemployment and balance of payments equilibrium as the cost savings enjoyed by
large firms are needed to maintain our export competitiveness against other countries.

Level Descriptor Marks


1 Listing and superficial explanation of impact on society. Lack economic framework. 1-5
2 Well explained, balanced (both positive and negative) micro-economic impact of large 6-8
firms on society in terms of impact on consumers, allocative efficiency, productive
efficiency and some attempt with examples.
Year 6 Preliminary_9732 [Turn over]
RI 2012
22

3 Well explained, balanced (both positive and negative) micro-economic impact of large 9-11
firms on society in terms of impact on consumers, allocative efficiency, productive
efficiency; elaborated with good examples.
AND
Good explanation of macro-economic impact.

E2 Supported judgment on whether large firms are preferred over smaller ones in 3-4
globalised world; need to have balance of micro and macro aims.

E1 Judgment, with weak reasoning, on whether large firms predominance is preferred in an 1-2
economy in globalised world.

3 The new Gardens by the Bay opens up 101 hectares of tropical foliage to the public and entrance to
the outdoor garden is free. It would be an icon all Singaporeans can be proud of and bring to mind
when foreigners talk about Singapore. The Gardens are also part of the lungs of the Singapore
ecological system.

a. Analyse why government intervention may be needed in the markets for public goods and for
goods which generate positive externalities. [10]

b. Discuss whether direct government provision of the Gardens by the Bay is the most appropriate
means of achieving an efficient allocation of resources. [15]

a)
Public good
A pure public good will display the characteristics of non-rivalrous in consumption and non-excludability:
Explain how these 2 characteristics will result in zero provision of the good if left to the free market.

Non-excludability: Once the good is provided, it is difficult or impossible to exclude non-payers from
consuming the good. The problem of free rider will accompany this characteristic, ie no rational consumer
will reveal his effective demand for the good nor pay for the good. Therefore, no profit maximizing firm will
supply this good in the free market. Eg, in the provision of more greeneries (lungs) in Singapore to generate
cleaner air and to maintain a healthy eco-system, non-paying individuals cannot be excluded from benefitting
from the clean air and healthy eco-system in Singapore.
(Note: it is not acceptable to explain non-excludability in terms of entry to the gardens. Free entry does not
mean non-excludable. There are only a few entry points to the gardens and the govt can easily set up
barriers at those few points.)

Non-rivalrous in consumption: The provision of the good to an additional user will not lower the
satisfaction of existing users consuming the good. The marginal cost of providing the good to an additional
user is equal to zero. For allocative efficiency to prevail, the price of the good must be equal to the marginal
cost of the good. However, in the free market, no profit-maximising firm will price his good at zero dollars to
achieve allocative efficiency. Any non-zero price charged will thus be allocatively inefficient, leading to
market failure. (Eg, explain how clean air and healthy eco-system is non rivalrous in use.)

Govt provision is necessary because consumers will be better off with the production of clean air and healthy
eco-system.

Positive externalities
(Positive externalities can be generated either from consumption or production of a good. Students should
provide rigorous explanation using one of the above and use an example to illustrate well. Many tried to do
explanation simultaneously for externalities generated from both consumption and production and the result
is a confused and incoherent essay.)

If left to the free market, individuals will consume base on pursuit of self interest, ie will look at his private
cost and private benefits only in deciding how much of a good to consume. However, for some goods, there
is a divergence in the private benefits and social benefits of consuming the good, ie there are external
Year 6 Preliminary_9732 [Turn over]
RI 2012
23

benefits to third party in society which the individual will not take into consideration in making his
consumption decision. This leads to an under-allocation of resources to that market and therefore market
failure. Using the preamble :
- Private benefits of visiting the garden : walk/jog and fresh air leads to maintenance of physical and
mental well-being. (accept other possible benefits :family bonding and well-being or personal pride of
Singaporean.)

- External benefits of visiting garden includes a healthier and productive labour force, increase in
output and services if labour force is healthy, lower financial burden on govt due to medical cost
(assuming government subsidises or provides healthcare for free). (accept other possible external
benefits :greater social stability with cohesive families and strong Singaporean national pride)

- From the governments standpoint, healthy individual, family well-being, social cohesion and national
pride is deemed desirable for society and is underachieved if left to the free market. To the govt,
welfare to society is not maximized, leading to market failure and therefore the need for it to
intervene.

Using the MSB/MSC framework given by the following diagram Fig 1:

Cost/Benefits
MPC = MSC

C
B

A
MSB

MPB

0 Qe Qs
No. of visits to the Gardens

Private individuals will be motivated by self-interest and the consumption decision made will be based on
MPB = MPC at Qe.
The socially optimum level of output is achieved when MSB = MSC, at Qs.
The under-usage of gardens (Qs- Qe) leads to a dead weight loss of area ABC.
Thus, left to the free market, visits to gardens will be below the optimal level and this misallocation of
resources by the free market provides a case for government intervention.

Level Descriptor Marks


1 Superficial explanation of public goods and positive externalities and how they lead to 1-4
market failure.
2 Good explanation (but allow for some gaps) of public goods and positive externalities 5-6
and how they lead to market failure. Some use of economic theory/ framework in
explanation.
3 In-depth explanation of public goods and positive externalities and how they lead to 7-10
market failure. Good use of economic theory/ framework in explaining public goods and
positive externalities and good use of examples to illustrate. Note: allow max 10m mark
for an excellent answer which may not use example from the preamble.

Examiners comments:
1) Poor use of examples in the illustration of market failure. For positive externality, there is inadequate
development of what the marginal ext benefits (MEB) are, third parties who are affected and how they are
affected. This may be due to the inability to first identify the source of the externality, whether it is generated
Year 6 Preliminary_9732 [Turn over]
RI 2012
24

by production or consumption.

2) Some students are very confused between under-consumption of goods which generate positive
externalities and under-consumption of goods due to imperfect information. The result is lengthy discussion
which is mainly incoherent. The idea of self-interest motives did not come through well in explanation of how
market failure arises.

3) Poor development of non-rivalrous concept . Non rivalrous does not mean marginal cost of producing the
good is zero, but the marginal cost of providing to an additional user is zero. Also, the concept of non- rivalry
means that for allocative efficiency, a good has to be priced at P=MC = 0. But no profit maximizing firm will
do that. The proper conclusion to this characteristic should be that firms will not charge a price of zero but
any non-zero price charged will be allocatively inefficient.
(The conclusion should not be that no firm will produce this good in the free market.)

4) Students should highlight why a missing market (in the case of public good) is a market failure. Such
goods are beneficial to society but would not be produced due to the 2 characteristics of non-rivalry and
non-excludability.
(b)
If explanation of market failure has been done using given context of Gardens by the Bay in part (a), no need
to repeat explanation of how market has failed in part (b). However, reference need to be made explicitly to
part (a)

Thesis: provision by govt due to an identified source of market failure.

[NOTE: Explanation of any one of the following source of market failure as basis for provision is sufficient to
score full credit in thesis.]

(i) On basis of positive externality generated from consumption

If left to free market, there will be under allocation of resources because the private individual will only look at
his private cost and benefit in making a decision to visit the gardens. Referring to part (a), private
individuals will be motivated by self interest and the consumption decision made will be based on MPB =
MPC at Qe. The socially optimum level of output is achieved when MSB = MSC, at Qs.The under-usage of
gardens (Qs- Qe) leads to a dead weight loss of area ABC.
Thus, left to the free market, visits to gardens will be below the optimal level and this misallocation of
resources by the free market provides a case for government intervention.

From the Singapore governments standpoint, healthy individual, family well-being, social cohesion and
national pride is deemed desirable (in fact some would say critical) for society and is grossly underachieved
if left to the free market. In fact the govt would say that these benefits are valueless.

In consideration of the extensive positive benefits and the difficulty of putting a monetary value to them,
providing free to consumers could lead to better allocation. This encourages more visits and the external
benefits (in terms consumption) is healthier population, lower medical burden on govt and a more productive
labour force, as well as promotion of family cohesion, national pride and identity icon.

(ii) On basis of positive externality generated from production

In the given context, it is likely that there are external benefits from production of the gardens in terms of
spillover effects to tourist related industries. The iconic gardens may attract foreign visitors and more tourists
visits has spillover effects on the other tourist related industries like hotel, transportation, restaurants. This
may lead to increase in employment and economic growth.
(Private cost is cost of producing the gardens and private benefit is the revenue generated from ticket sales
to the gardens)

(iii) On basis of zero marginal cost in providing for one more visitor

Entry to the gardens is excludable but usage is non-rivalrous. Since marginal cost of providing for one more
Year 6 Preliminary_9732 [Turn over]
RI 2012
25

visitor is zero, the govt chooses to allow free entrance for allocative efficiency (P=MC).

(iv) On basis of a public good

Here, the benefits need to be explained in terms of maintenance of healthy ecosystem and provision of clean
air by the greeneries (lungs)
Non-rivalrous (does not reduce benefits to the next user) and non-excludable (cannot prevent a non-payer
from benefitting from the healthy ecosystem and from breathing in clean air).
No profit maximizing firm will provide for this service which will increase the societys welfare.

Anti-thesis

(i) Limitation of govt direct provision for free:


Civil servants are not profit motivated and therefore may not be cost conscious enough to achieve productive
efficiency. Given budget for public projects are often busted. Civil servants or politicians may be more
concern about the popularity votes and therefore over provide more than optimal. This may lead to waste of
resources or huge strain on govt resources as govt expenditure may be way higher than the benefits to
society.
Conversely, if a tender is awarded to a private firm for such similar construction, the firm must stick to its
obligations by minimizing its costs.

(ii) Alternative measures:


Private firm produce but subsidy given:
As discussed earlier, if the source of failure is due to external benefits, and if the provision of the gardens is
left completely to the free market, an under-allocation of resources will result. So a possible solution may be
to leave to the private firm to produce but govt intervene in the form of subsidy. Subsidy given will result in
greater production and lower entrance fee. More Singaporeans will visit the gardens and frequently too.
Greater productive efficiency may be achieved because firms are profit motivated and need to minimise cost
of production.
(accept other relevant points :as one of many places of interest in Spore, the gardens have to compete for
the Singaporean and tourist dollar. Dynamic efficiency may be achieved too as firm need to innovate and
improve services available to remain competitive.)

OR

Govt construct infrastructure and let private firm operate:


It is unlikely that any single firm is able to undertake the construction of such a mega project which requires
huge resources in terms of funds (S$1 billion) and vast land. It may be possible for the govt to construct the
infrastructure and thereafter tender out to a private firm to operate (like in the case of the MRT in Singapore).
Here, the govt absorbs the cost of the infrastructure and the firm only incurs operational costs. Therefore the
entrance fee charged is likely to be low enough to encourage more visits by Singaporeans. Again, the
element of profit motivation in private firms is likely to encourage greater efficiency in the running of the
gardens.

[If other measures like campaign or education are recommended, the link to the question must be clearly
explained. For example, the subsidy alone may not be effective to increase visits to the Gardens because
Singaporeans need to be informed of the benefits of visiting parks too. Campaign and education should not
be abruptly introduced as an alternative to direct provision.]

Possible Judgement
(accept any others not listed below but which are sound in the given context):

i) Possible govt failure- The free market may under-allocate resources to the gardens. However, direct govt
provision of the gardens may not achieve efficient allocation of resources either in the given context because
the govt does not have perfect information and it is extremely difficult to put a monetary value to all the
externalities in the given context .Intervention in the form of subsidy to private operators or consumers is
equally difficult to estimate. Any form of govt intervention can at best, only lead to an allocation that is closer
to the efficient level.
Year 6 Preliminary_9732 [Turn over]
RI 2012
26

ii)It seems that govt provision is the most appropriate because it is unlikely that the private sector can
undertake such a mega project and there are too many external benefits (both from consumption and
production). In particular, the social ones which the govt may feel to be too critical to be left to the private
sector.

iii)The govt intervention here may not be for efficiency purposes but for the purpose of increasing the quality
of life of Singaporeans.

Level Descriptor Marks


1 Listing and superficial explanation of how market fails in given context and 1-5
therefore a need for govt intervention.
2 Good explanation of how market fails in given context and therefore a need for 6-8
govt intervention. (credit to be granted if explanation has been done in part(a)).
Balanced answer on how direct govt provision leads to efficient allocation of
resources. Explanation is somewhat clear and applied to given context.

3 Balanced answer on how direct govt provision leads to efficient allocation of 9-11
resources. Explanation is clear, thorough and in-depth and is well applied to
given context

E2 Insight or supported judgment on whether direct govt provision is most 3-4


appropriate or otherwise.

E1 Judgment, with weak reasoning, on whether direct govt provision is most 1-2
appropriate or otherwise.

Examiners comments:

1) The thesis to this question is missing in many scripts.


- Note that if the source of market failure has been explained in part(a), there is no need to repeat in part (b)
BUT students need to make reference explicitly to part (a). Many scripts did not make such references to the
answer in part(a) at all, especially to the diagram drawn in part (a).

- Many scripts went straight into explaining how govt provision is not suitable (anti-thesis) without first
identifying the source of market failure and then explaining how govt provision may correct the given market
failure.

2) Many attempted thesis were poorly done too.

The main problem is that students tried to discuss all possible sources of market failure simultaneously in
this given question. Externalities generated from production are confused with externalities generated from
consumption and these are in turn discussed together with market failure due to public good.
An example of a confused answer:
The gardens is a public good that generates positive externality. Third party benefits include a healthier
society, fresher air for the nearby areas, and the increase in the number of tourists to Singapore. Hence if left
to the free market, it will be under-consumed. The govt could use alternative measures like education or
legislation to force more Singaporeans to visit the gardens
Students are reminded that any source of market failure should be discussed while assuming that there are
no other sources of market failure. Other sources of market failure may be brought up for discussion as
insights or evaluation or in anti-thesis, depending on the context.

3) Anti-thesis is also poorly done by many students.

- Many failed to explain the limitations of govt provision before introducing alternative measures.

- Alternative measures are thrown in blindly without considering the context carefully. An example of a poor
Year 6 Preliminary_9732 [Turn over]
RI 2012
27

analysis:
Instead of direct provision, the government should set a low price and provide subsidy. This will increase
consumption to socially optimum level.
The above analysis simply doesnt make sense especially when comparing to a consumption level that is
fully subsidized, (with price = zero under direct provision)!

- When other measures like campaign or education are recommended, the link to the question is non-
existent! Campaign and education should not be abruptly planted as an alternative to direct provision.
Students are reminded that such links must be clearly explained. For example, govt provision alone may not
be effective to increase visits to the Gardens because Singaporeans need to be informed of the benefits of
visiting parks too.

- The context of this question was difficult to get around for most students, thus seeing many generic and
theoretical arguments of subsidies, regulation, education and campaigns with little application. Legislation to
force students to visit the gardens does not make sense in the given context. Some students went into trivial
details like how to raise funds for the operation of the gardens, some went into macroeconomic problems
(example: large funds required to provide the gardens borrowing from private sector increase demand for
loanable funds push up interest rates reduce investment and thus LR productive capacity and affects
future SOL!).

4) A common error - Gardens by the Bay is not a public good and therefore the government should not
directly provide. Such students failed to see that there is government direct provision in the areas of
education and healthcare, services which generate significant positive externalities in consumption.

5) Area for improvement In the evaluation of possible govt failure, students identify that it is difficult to
calculate accurately the MEB but did not move on to elaborate on why that is so. They should move on to
explain, for example, that it is difficult to quantify the monetary value of cleaner air, or to put an accurate
value on the additional tourist expenditure due to the gardens.

4 The Singapore government has tightened the supply of foreign labour. In the short term, this may
mean forgoing business opportunities and accepting slower growth.

Discuss the conflicts in the government objectives that may arise from the use of this policy to
achieve productivity-driven growth. [25]

Suggested Outline:
Introduction:
Governments objectives include micro and macro objectives.
Micro objectives include: greater allocative and productive efficiency, reduce income inequality and
greater income equity.
Macro objectives include: internal stability such as sustainable economic growth, low unemployment,
price stability & external stability such as exchange rate stability.
The aim of tightening the supply of foreign labour policy is to achieve productivity driven growth. The
policy is to cut down the number of unskilled or semi-skilled foreign cheap labour in Singapore. Affected
industries are mainly the labour intensive types such as cleaning companies, transport companies, F&B
industry

Conflicts
- Fall in actual growth.
When there is a decrease in the supply of foreign labour, domestic firms will have to employ the more
expensive domestic labour when the PED for labour is price inelastic, this leads to an increase in cost of
production. Firms will cut down on production which is illustrated by an upward or leftward shift of the AS
curve.
At the same time, the demand for goods and services by the foreign labour will decrease, causing C to
fall. FDI especially in labour intensive industries will decrease due to the fall in supply of labour. Both C
and I decrease leading to a fall in AD.

Year 6 Preliminary_9732 [Turn over]


RI 2012
28

GPL AS2

AS1
P2
P1

AD2 AD1

Y2 Y1 Real NY
These changes lead to a fall in real NY from Y1 to Y2 in short run, actual growth decreases.

- Increase in unemployment
Since the demand for resources is a derived demand, when there is a fall in real NY, the unemployment
rate will increase.

-Price instability
With the increase in COP for the firms due to the more expensive domestic labour, this will cause wage
push inflation. Alternatively, if AS is more than AD, the GPL will rise from P1 to P2, hence leading to
loss of price stability.

-BOP
With the fall in FDI, this may worsen the capital and financial account. But since there will be less foreign
workers here, there will be less repatriation of wages back to their hometowns, therefore improving the
current account.

No conflicts
-Increase in potential growth
When the firms experience anCOP, they may choose to automate, mechanize and re -organize in the
long run. They can also send the workers for more training to increase their productivity. These enabled
the productivity driven growth causing an increase in productive capacity in the economy, illustrated by
an outward shift of the LRAS curve.

LRAS1 LRAS2
GPL

Real NY
Yf1 Yf2

With this increase in productivity, FDI may once again be attracted to come back, bringing about an even
further increase in potential growth in the long run.
Alternatively, the increase in FDI will increase AD, thus will bring about actual growth in the LR.
Sustainable economic growth can be attained.

-Decrease in unemployment
In the short run, when the PED for labour is price inelastic, more domestic labour will be employed when
the foreign labour supply is tightened.
In the long run, with the sustained economic growth attainable, more resources will be employed.
Alternatively, if productivity growth is achieved through automation/mechanization, less labour will be
employed.
Year 6 Preliminary_9732 [Turn over]
RI 2012
29

-Price stability
With an LRAS, assume AD remains constant; it will lead to an improvement in price competitiveness of
our goods and services.

-Income inequality will be reduced


When the supply of foreign labour is tightened, the supply of unskilled & semi-skilled workers (domestic +
foreign workers) will be decreased, illustrated by a leftward shift of supply curve from S2 to S3. If
demand for such workers remains unchanged, the wage level of the domestic workers will increase from
W2 to W3. This helps to reduce the income gap between the low income group and the high income
group.
Wage
rate
S1 (domestic workers)
S3(domestic + foreign workers)
S2(domestic + foreign workers)
W3
W2

Qty of labour
Conclusion:
-The policy may lead to short term conflicts in objectives, but in the long run will be compatible with many
of the governments objectives.
-This reduction on the reliance of foreign workers to promote productivity growth may be a more
sustainable measure. Since the firms have no choice but to think of all ways to replace the cheap
foreign labour. The automation and mechanization ways will definitely enhance productivity growth to be
attained and the economy will benefit in the long run. Governments policies such as subsidizing the
trainings of the workers through SPURS for eg, will reinforce this producutivity growth to be attained in
the long run.
-The cheap foreign labour can be more costly in the future, for eg, Indonesia govt had imposed a
minimum wage level on the maids coming to Singapore. Rather than to increase our reliance on
external supply and subject ourselves to the uncertain factors, it is better to look at internal factors and
support this productivity driven growth.
-In the short run, it looks like the group of domestic unskilled and semi-skilled workers is the only one to
gain in terms of higher wage level. But in the long run, the economy will benefit in many ways as
explained in the answer, all will stand to gain.

Mark scheme
Knowledge, application, understanding, analysis
L1 Lack use of economic framework/concepts in analysis 1-9
Lack scope and depth- one sided answer only
Irrelevant answer
No attempt to link to productivity driven growth
L2 Use of AS/AD framework/concepts in analysis 10-14
Explanation of concepts is underdeveloped or lacks rigor
Made attempt to link to productivity driven growth, develop
conflicting and compatible attainable macro objectives only.
L3 Good use of AS/AD framework in analysis 15-21
Good application to the Singapore economy
Cover productivity growth, cover both conflicting and compatible
attainable macro and micro objectives
Evaluation
E1 An unexplained judgment , one that is not supported by analysis 1-2
E2 Evaluative assessment supported by economic analysis 3-4
Excellent synthesis and is able to justify and arrive at a convincing
stand
Year 6 Preliminary_9732 [Turn over]
RI 2012
30

Examiners comments.
1. Many students could identify the causes for theAD and AS, but fail to combine them to look at the net
effect on real national income and price level. Instead they deduced these effects from the two
scenarios separately. Though it is not critical, but it will improve the presentation.

2. Many students left out the micro objective on income equity. Those who covered macro objectives also
tend to include the secondary effects when they discussed the BOP position. For example, after the
answer deduced that in the SR, there might be cost push inflation, the students will go on to explain that
the increase in the price of our exports will lead to a fall in its demand, assuming that the PED of our
exports being more than one, therefore export value will fall. While import value remains unchanged,
this will worsen the current account in the BOP. This effect on BOP did not stem from the fall in supply
of foreign labour but it was from inflation, therefore such secondary effects should not be included.

3. Many students did not see that the question implied that the fall in supply of foreign labour will push firms
to be creative in their ways to reduce their cost of production. Instead, many wrote on governments
policies such as government subsidizing the trainings of workers, government reducing interest rate to
enable firms to make investment on capital goods. Though this is not critical, but it showed that the
student is not aware of what is going on in our economy.

5 The Euro zone is a key market for Chinese exports and the economic woes in the Euro zone has
resulted in the slowing of Chinese exports to the region. This has led to concerns about a slowing
down in Chinas economic growth.

Assess the range of policies that might be appropriate for China to focus more on domestic sources
to drive its growth. [25]

Suggested Outline:

Introduction

Briefly explain that for China to focus more on domestic sources to drive economic growth means a shift
away from an over-reliance on external sources of growth (i.e X, inward FDI)

Clarify the meaning of domestic sources of growth: Economic growth that results from a rise in internal
demand: Increase in C, domestic I and G.

State that China can consider the following range of economic policies:

Demand-Mgt Policies; MP, FP


Exchange Rate Policy
Supply Side Policy
Trade Policies

Body/Analysis

Explain how domestic sources allow China to achieve sustained economic growth

Increase AD via increase in C, domestic I and G to achieve actual growth

Briefly explain the k effect and illustrate with AD-AS diagram

Increase SRAS to achieve actual growth

Increase LRAS (via domestic I, G) to achieve potential growth

Year 6 Preliminary_9732 [Turn over]


RI 2012
31

Explain how the range of policies would allow China to influence the domestic sources of growth to achieve
sustained economic growth.

1) Demand Management Policies

Expansionary MP: reduce i/r, increase Ms

Chinas Central Bank can attempt to reduce interest rates to stimulate domestic C and investment. With a cut
in interest rates, this causes a fall in borrowing costs, raising domestic C especially on big-ticket items (or
reduces opportunity cost of C). Alternatively, a lower interest rate also discourages savings, which
encourages C.

Similarly a cut in interest rates also lowers the borrowing cost for domestic firms in financing investment
projects. With a higher expected rate of return from investment, more investment projects become profitable
and hence raises the incentive for domestic investment (Illustrate using MEI diagram).

In addition, the Chinese government can also make efforts to ensure better access to such loanable funds
for non-state owned enterprises or non-SOEs (SOEs accounted for more than 53 per cent of non-agricultural
fixed investment while employing only 13 per cent of the total workforce.)

Hence expansionary MP have the effect of raising AD and achieving actual growth.

Expansionary FP: increase G, reduce (direct) Taxes

Increase in G can be centered on improving welfare assistance especially for the lower income group
(higher MPC to induce a larger k effect).

Reduction in direct taxes in the form of both personal income and corporate taxes to stimulate domestic C
and I.

With a lowering of personal Y taxes, this increases households disposable income increase in
autonomous C

By improving efforts to redistribute income, China can aim to narrow to income gap and raise the social
mobility of the sizable middle-class to effectively raise C.

With the increase in G, I, C, expansionary FP have the effect of raising AD and achieving actual growth.

Limitations of demand-management policies:

- Ability to stimulate C and I depends on having both favourable consumer and business sentiment. In the
situation of poor animal spirits, lower interest rates/direct taxes will do little in raising C and I due to
households being cautious about spending and firms expecting a weak profit outlook tend to withhold
spending on investment projects.
- Ricardian equivalence effect of lowering household taxes may actually encourage savings.
- Opposing income and substitution effects of tax cuts may discourage work effort.
- DD-pull inflation, conflicts with price-stability objective.

2) Exchange Rate Policy

China can allow a further revaluation of the Yuan to reduce excessive export-led growth and give impetus for
domestic producers to switch to producing more for more the domestic market. This would in turn raise
domestic C and hence raising AD and achieving actual growth.
Year 6 Preliminary_9732 [Turn over]
RI 2012
32

Additionally, with a stronger Yuan, this raises the price of Chinese exports in foreign currency terms, lowers
the price of M in domestic currency terms. Assuming M-L condition is satisfied, this leads to a worsening of
BOT, reduction in net X.

Limitations of Exchange Rate Policy:

M-L condition may not be satisfied in the SR due to issue of fulfillment of trade-related contracts etc, the
observed worsening of BOT may not take place immediately.

Further, not all domestic producers may be able to simply and easily switch from producing for the foreign
market to cater to the domestic market due to the nature of the goods produced. Any further revaluation of
the Yuan should be calibrated in a gradual and modest manner so as give sufficient time for domestic
producers to make the necessary structural adjustments.

3) Supply-Side Policy

China can pursue a mix of both market-oriented and interventionist supply side policies. In terms of market-
oriented policies, this can be in the form of deregulation and privatisation of previously government-owned
industries. By doing so, firms within such industries when subjected to market forces have the incentive to
raise their cost-efficiencies and compete better against their foreign-counterparts. (With lowering of COP,
raise SRAS to achieve actual growth)

Overtime, if successful, they will be able to build up sufficient capabilities to gain market share and undertake
higher levels of private sector investment (contributing to actual and potential growth via accumulation of
capital goods through greater domestic I).

In terms of interventionist policies, subsidies and tax breaks or concessions can be given to aid private
domestic firms to lower their cost of production and/or raise their profit potential. This also encourages higher
levels of private sector investment (contributing to actual and potential growth)

(Supply-side policies can also be discussed in relation to the raising of productivity in the Chinese labour
market or infrastructure development)

Limitations of Supply-Side Policy

- Not all industries can be deregulated or privatised due to strategic reasons.


- Vested-interest by certain power interest groups may lobby against the break-up of SOEs, thus
hindering the deregulation/privatization process.
- Some firms may lack sufficient internal economies of scale or access to sufficient financial resources
to compete against the stronger foreign competitors
- Being-profit driven, firms may have incentive to monopolise market and raise prices reduce
consumer surplus and welfare.
- Subsidies, tax concessions once given may be subjected to lobbying in the future and difficult to
withdraw.

4) Trade Policies

This can take the form of protectionist measures to shelter domestic industries from foreign competition and
act as a form of expenditure-switching policy.

Eg. With use of an import-tariff (Illustrate with tariff diagram), this raises the price of imports and improves the
ability of domestic producers to compete better against foreign producers. At the same time, domestic
consumers are also encouraged to switch to relatively cheaper domestic substitutes increase in domestic
Year 6 Preliminary_9732 [Turn over]
RI 2012
33

C increase AD raise actual growth.

Limitations of Trade Policies:

- Such expenditure switching policies assume that domestically produced goods are good substitutes
for foreign imported goods which may not be the case due to differences in product quality and
variety.
- M tariffs protects cost-inefficient domestic producers, it goes against the Principle of Comparative
Advantage, allocatively inefficient as resources are not directed to where they are most highly
valued, likely to invite retaliation.

Conclusion/Synthesis

- For China to successfully to switch from a growth that is more externally-driven to one that relies
more on domestic sources, it will require an appropriate policy mix in view of the various policy
limitations and the policy conflicts that may arise. Certain policy measures like protectionist
measures to encourage expenditure switching should at best be a short-term measure and should
not be pursued in the long-run. The rebalancing of the Chinese economy is a long-run process and it
requires a careful calibration of welfare-reforms to raise the middle-class as well as economic
incentives to grow the domestic sector.

Knowledge, Application, Understanding, Analysis


L1 For an answer that is descriptive and lacks the use of economic 1-9
framework/concepts in analysis
Glaring conceptual errors
One-sided answer, limitations of policies not discussed.
Lacks application to the Chinese economy context (no mention of Chinese
economy, i.e. purely theoretical answer) Max 9
Incorrect/poor grasp of question, does not address domestic sources of
growth but instead analysis focuses on how economic growth take place.
L2 Use of appropriate economic framework/concepts in analysis AD-AS 10-14
diagram etc
Sufficient scope of coverage
Balanced answer that attempts to discuss the limitations of the
policies
At least 2 distinctly different policies (eg. M.P, F.P) discussed
If only 2 policies discussed Max 14 marks
If domestic sources of growth is linked only to actual or potential
growth Max 14
Sufficient depth of analysis
Some application to Chinese economy context (e.g. size of K for China
etc)
Limitations discussed but not specific to the policies.

L3 Good use of economic framework/concepts in analysis AD-AS diagram 15-21


etc
Good scope of coverage able to link domestic sources to both actual
and potential growth.
At least 3 policies discussed.
Max 18 if only 3 policies discussed.
Good depth of analysis.
Good application to Chinese economy context.
At least one of the policies have limitations that are specific to it.

Year 6 Preliminary_9732 [Turn over]


RI 2012
34

Evaluation
E1 Unexplained Judgement that is not supported by economic analysis 1-2
E2 Evaluative assessment and judgement supported by economic analysis. 3-4
Excellent synthesis in which the student is able to justify the use of
appropriate policies given the Chinese context and arrive at a convincing
stand (based on earlier arguments and counter arguments)

Examiners Comments:
This question required students to consider carefully the range of policies needed for China to shift to a more
domestic driven source of growth. On the whole, it was pleasing to see that the majority of candidates were
able to grasp the notion of domestic sources of growth and relate it to the use of appropriate policies and
thereafter explain the effects on achieving sustained growth. The best approaches to this question firstly
explained the meaning of domestic sources of growth, i.e domestic C, domestic I and G. This was usually
followed by a consideration of demand management policies like expansionary FP and MP to raise C, I
and/or G to raise actual growth, as well as Supply-side policies to raise potential growth. The highest L3
mark range was reserved for candidates who went on to also consider the use of other appropriate policies
like trade policies to encourage consumption of domestically produced goods. Some candidates were able to
demonstrate a good understanding of the constraints facing the Chinese economy and hence show good
application to context. Insightful comments included a critique of the weak social security support which
tends to promote a high level of private savings, or the need to enhance the set of existing income
redistribution policies to raise the social mobility especially of the large segment of the middle class in China
to promote higher domestic consumption. Such responses were well rewarded apart from the more standard
demand and supply-side policies explanation.

Weaker responses on the other hand did not demonstrate clearly a good grasp of question requirement and
included for example irrelevant discussions of how a depreciation or weakening of the domestic Chinese
currency would allow China to raise export competitiveness or that a lowering of interest rates or corporate
tax rates would attract FDI.

A common weakness observed in a number of scripts was the lack of clear links to actual and/or potential
growth within the policy discussions. For instance, for the explanation of demand management policies, after
the explanation of the multiplier effect, candidates would round off the discussion by stating that national
income increases more than proportionately. As such the link to economic growth was omitted.

In terms of fiscal policy, a common weakness is the lack of clarity when it comes to the point of tax cuts. A
clear identification of the type of taxes is needed, i.e. personal income tax or corporate tax. A number of
candidates included a discussion of cuts in consumption tax which was unnecessary and erroneous when it
was explained in terms of raising AD.

In terms of application to the Chinese economy context, a common point made by candidates was that the
Chinese economy has a large domestic sector and hence the multiplier effect is likely to be large. However,
there was no clear link to the MPW component. Similarly, some argued that the culture of thrift in China
would mean a higher tendency to save but again there was no attempt to explain clearly how the multiplier
size can be argued to be small.

Financial crowding-out effect was an oft-cited point in relation to the limitations of expansionary FP.
However, a number of candidates explained that with an increase in G, there will be a reduction in the
availability of loanable funds and hence a fall in the supply of loanable funds, when the correct explanation
should be an increase in demand for loanable funds driving up interest rates and hence deterring private
domestic investment.

Demand-pull inflation was also mentioned as a possible problem of expansionary FP or MP. While this was a
valid concern, this was not well-explained as a point in relation to the state of the Chinese economy. Most of
the responses seemed to suggest that demand-pull inflation is inevitable with some scant reference to the

Year 6 Preliminary_9732 [Turn over]


RI 2012
35

need to increase AS. Others unnecessarily went on to explain the secondary effects of demand pull inflation
on the economy.

Candidates are also reminded that the discussions of demand management and supply-side policies should
be discussed discretely. A significant number of candidates would start off with a demand management
policy and before explaining how the policy works on the demand side with sufficient depth, launch into the
probable supply side effects of that policy. For instance, a candidate cited FP as a possible demand
management policy to achieve economic growth but it was not well explained in terms of the effects in raising
AD and eventually actual growth. Then, this particular candidate unwisely wrote at length about the supply-
side side effects of a particular government project to improve on the building of infrastructure. A similar
observation applied to the explanation of MP where candidates explained that with a lower interest rate, an
increase in investment would increase productive capacity and hence raise potential growth, while the actual
growth effect discussion from the demand side was not well developed. Candidates should demonstrate a
better awareness of the primary purpose of demand management and its difference with supply-side policies
so that their analysis can be more focused and well directed. Since the question is on policy per se, the
candidates must deal with the different policies with sufficient depth and only attempt to comment on how in
the real world, government policies are not discrete and may impact both AD and AS simultaneously.

In terms of monetary policy, a common mistake was to confuse the money market with the loanable funds
market. Weaker candidates tend to explain that the lowering of interest rates happens when there is an
increase in the supply of loanable funds. The use of the MEI theory was also noticeably absent in explaining
the effects of lower interest rates on investment. Others went on to explain that a lower interest rate would
result in hot money outflows from the Chinese economy and hence worsen the BOP. This line of argument
however ignores the presence of capital controls imposed by the Chinese government.

For supply-side policy discussions, candidates can do better by elaborating on their points in terms of the
limitations of such policies. Often, candidates would simply state that supply-side policies have long time lags
or that the government would incur high costs. As such the opportunity to demonstrate development in
analysis as well as good application to the Chinese economy context was lost.

In the case of trade related policies, some candidates explained the use of import tariffs to facilitate import-
substitution and encourage greater consumption of domestically produced goods. While this was relevant in
terms of imports will be discouraged with imposition of tariffs but it should be noted that punitive tariffs could
not have been imposed on all goods and services but at the most on a few strategic industries by a country.
The tariff protection argument is basically a partial equilibrium analysis. Thus, the argument for imposing a
tariff on a good so that domestic firms in a selected industry can become a source of growth is not very
sound since the tariff is with respect to a particular industry and is thus at the micro level. Moreover, most
candidates do not make the link between tariff imposition to growth itself, stopping at increasing domestic
production of a good X. In addition, the analysis that followed often seemed to veer into a discussion of the
pros and cons of import tariff as a protectionist measure. The focus on domestic source of growth was thus
unclear. Candidates should also be reminded to avoid using non-standard abbreviations like govt, SSP,
EMP etc.

6 Trade of some emerging economies like China and India have seen stellar increases in the last
decade.

a. Explain the factors that may have led to rising trade of emerging economies. [8]
b. Discuss the view that emerging economies have more to lose than gain from globalisation.
[17]
Suggested Outline:
(a)
Introduction
Explain the characteristics of emerging economies- they are the developing economies which are
rich in resources, labour is mostly unskilled or low-skilled, rising technology level, going through
industrialization phase at the moment. These countries CA usually lie in low value added production,
eg China exports low value manufactured goods.

Year 6 Preliminary_9732 [Turn over]


RI 2012
36

Factor: Rising middle income group in emerging economies


As the emerging economies such as China and India are going through the industrialization phase,
capitalizing on their cheap labour and other resources and hence exporting low value added
manufactured goods such as textiles and agricultural goods. These labour intensive industries
increase the employment opportunities within the emerging economies and hence increase the
income of their people. The increasing purchasing power of the rising middle group demand for
higher value added goods such as air-con units, refrigerators from the other emerging economies.
Hence, this has increased trade among the emerging economies.

Factor: Removal of trade barriers among the emerging economies and signing more FTAs with
developed economies
When the governments of the emerging economies sign FTAs with each other (e.g. China-ASEAN
FTA) to remove trade barriers such as tariffs imposed against their exports, this improves the price
competitiveness of the exports in the foreign countries. The demand for emerging economies
exports hence increase. Similarly, as emerging economies remove tariffs on imports, import
expenditure increases. Increases in both exports and imports enhance trade among the emerging
economies. The same goes for FTAs between emerging economies and developed economies (e.g.
EU-Peru FTA), the impact is even greater as the developed economies import such low value added
goods to produce their re-exports.

Factor: MNCs entering emerging economies


As emerging economies like China and India relax the investment regulations by foreign companies,
many MNCs outsource their production into these emerging economies to take advantage of their
cheaper labour and other resources. This helps to lower the cost of production and increase the
profits of the MNCs. Eg, Manufacturing firms setting up production plants in China while IT
companies setting their base in India. These goods will be exported from these emerging
economies, hence increases trade.

Factor: Better infrastructure in emerging economies


The governments of these emerging economies pour in large amounts of funds in improving their
infrastructure such as building bigger ports, widen roads to improve the efficiency of the transport
system, improving the telecommunication system and building more electric plants to meet the rising
demand. Eg China developed the coastal areas to facilitate trade. Such development will also attract
more MNCs wanting to make them the base centers to reach out to the other regional Asia countries.
Similar to above, the increased production activities by MNCs in emerging economies increases
trade.

Factor: Decreased transport costs


Improvements in technology have reduced transport costs (e.g. containerization, more fuel-efficient
ships and planes). Since transport costs prevents trade by reducing the gains from trade (Law of CA
assumes no transport costs), a reduction of transport costs increases the benefits of trade and
hence, encourages greater trade.

Conclusion:
It is a combination of factors that enhance trade of the emerging countries. They are able to gain
from trade.

Mark scheme
Knowledge, application, understanding, analysis
L1 Inadequate development: listing of points 1-3
Journalistic style of writing with no economic concepts
L2 Lapses in explanation and scope of coverage 4-6
Cover 2 factors
L3 Cover at least 2 factors and explain them in depth linking to the 7-8
characteristics of the emerging economies

Year 6 Preliminary_9732 [Turn over]


RI 2012
37

Examiners comments:
To secure a level 3 for Part a), the candidate must explain and illustrate how certain factors lead emerging
economies to engage in more trade. Surprisingly, few candidates bothered to explain how economies are
classified as emerging economies. Instead, many candidates launched straight into explaining the Law of
Comparative Advantage and claimed that it was a factor that lead to increased trade. That is incorrect. The
Law of Comparative advantage provides the basis of trade (why countries trade) but is insufficient to fully
explain increasing trade in the last decade. A better approach would be to link how a fall in transport costs
and the removal of trade barriers to changes in the production pattern of emerging economies that lead to
more trade between economies. Emerging economies, in an attempt to adapt to the demands of multi-
national corporations in todays much more integrated supply chains, find new niches in producing goods or
services. For example, due to outsourcing of parts (of cars, shoes, phones, etc.) to off-shoring call centres
abroad, MNCs now enable emerging economies to specialise according to their CA.

Most candidates could identify proliferation of FTAs as a possible reason that lead to rising trade but failed to
provide the economic analysis of how a reduction in tariffs would increase trade. It must be emphasized that
candidates must remember that economic rigour can only be displayed through presentation of ones
analysis. An answer on globalization in the Economics examination must use the technical terms and tools
economists use and a journalistic style of writing must be avoided at all cost. Superficial analysis can be
avoided if illustrations specific to emerging economies were used to highlight the real world context of
emerging economies. The best answers provided excellent examples and were duly rewarded.

Suggested Outline:
(b) Introduction
Define Globalisation: Growing economic interdependence of countries worldwide through increasing
volume and variety of cross-border transactions due to freer trade in goods and services, freer flows
of international capital and greater mobility in the movement of labour. Brought about by
technological advancements, FTAs etc.

Gains and losses from globalization stems from the following aspects of globalization:

Freer flow of trade in goods and services


Freer flow of capital
Freer flow of labour
Technological transfers

State that the effects can be examined in terms of the macroeconomic and microeconomic effects.

Thesis: Emerging economies have more to lose from globalization.

Freer flow of trade in goods and services

- With the lowering/removal of trade barriers on imports, a fall in import prices, increases in
the quantity demanded for imports. This will lead to competition for import-substitution
industries fall in DD for domestically produced goods. This may lead to structural
unemployment in import substitution industries as the industry declines and workers who are
subsequently laid off find their skills unsuitable for other growing industries.

- Increased vulnerability to external shocks


As emerging economies become more integrated with the global economy and become
more dependent on external demand for exports, they can also become more susceptible to
downturn in external demand especially from the developed world. Depending on the nature
of its exports, eg. Chinas exports of manufactured electronics goods to develop world will
fall in the event of a global economic downturn, eg. Euro-debt crisis, since such goods tend
to be income elastic in demand. (China being an important-node in the global supply chain).
The greater the dependence on exports, the greater the magnitude of the effects. Hence, the
economy tends to undergo larger cycles in line of changes in global conditions (increased
volatility).
Year 6 Preliminary_9732 [Turn over]
RI 2012
38

- With the lowering of trade barriers and an increase in X growth increased utilization of
resources for production resource depletion -ve ext (eg. pollution) lower non-
material SOL (especially for resource-rich emerging economies whose environment is
exploited by resource-scarce developed economies)

Freer flow of capital


- Inflow of FDI may crowd out domestic investments (explain increase in i/r) over-reliance
on FDI for economic growth more susceptible to international shocks.
- Worsening of current account in the LR due to repatriation of profits.

Freer flow of labour


- Freer movement of labour will lead to brain drain, where local talents chose to work
overseas. fall in labour supply and more importantly, loss of productivity as fewer
innovative processes will developed fall in LRAS limits potential growth.

Anti-thesis: Emerging economies have more to gain from globalization

Freer flow of trade in goods and services


- Removal of trade barriers on an emerging countrys exports in the global market will enlarge
its export markets as exports become more competitive. This is because free trade will
enable countries to trade according to the theory of comparative advantage. As emerging
countries have a CA in cheap labour and tend to be resource rich, they are hence better able
to export low value added manufactured goods such as textiles to developed countries. This
will lead to an increase in DD for these countries exports in these sectors as they have a
lower opportunity costs in the production of these goods this will lead to an increase in
export revenue and rise in NX, cet par, rise in AD, NY and actual growth, employment. Rise
in NX also improves BOT, cet par, improves current a/c and BOP.
- Increase in trade between emerging economies: due to the development of global supply
chains, increase in exports of final manufactured gds from emerging countries such as China
to the developed countries would also benefit other emerging countries which supply the
intermediate inputs required for the manufacturing and assembly of the final product for
exports promotes exports-led growth of other emerging economies.

Freer flow of capital


- Freer capital flow will lead to FDI inflows. MNCs may outsource their production process into
these emerging economies to take advantage of their cheaper labour and other resources to
lower their production costs. This will lead to increase in investments in emerging
economies especially in those economies which are newly industrializing and experiencing
rapid economic growth (eg. BRIC economies). The net inflow of FDI AD and AS
increases NY increases (through the k-process) EG (actual and potential growth),
lower UN rate. There will also be to an improvement in the capital account and BOP in the
short run.
- FDI inflow will help emerging economies to develop its physical infrastructure and benefit
from technology transfer given that it is likely to be starting from a low tech-base. This will
help accelerate its rate of industrialisation and aid in sustaining economic growth over time.

Freer movement of labour may lead to increase in the supply of skilled and talented labour in
emerging economies as MNCs send skilled professionals to manage the local business
activities. There might also be reverse migration of skilled labour back home.
- Influx of foreign talent enhances quality of labour Increase in productive capacity
increase in LRAS potential growth
- Influx of foreign talent allows emerging economies to develop new niche areas especially in
diversifying the economy by allowing the economy to move up the higher value-added
manufacturing ladder, or move into the services sector in the future.

Conclusion: Overall, emerging economies can be said to have gained more than they have lost from
Year 6 Preliminary_9732 [Turn over]
RI 2012
39

globalization although the gains have not been even among the emerging economies. Fast growing
emerging economies such as the BRIC economies have been notable beneficiaries of globalization
due to favorable and conducive government policies that leverage on the benefits of globalization
while having in place appropriate policies to mitigate the costs of globalization.

Knowledge, application, understanding, analysis


L1 For an answer that is descriptive and lacks the use of economic 1-5
framework/concepts in analysis
Glaring conceptual errors
One-sided answer emerging economies loses or gains
Lacks application to emerging economies context Max 5 marks
L2 Use of appropriate economic framework/concepts in analysis AD-AS 6-9
diagram etc
Sufficient scope of coverage
T-A structure, at least 2 aspects of globalization discussed
If only 1 aspect of globalization discussed Max 6 marks
Ability to link to macroeconomic effects
Sufficient depth of analysis
Some application to emerging economies context.

L3 Good use of economic framework/concepts in analysis AD-AS diagram 10-13


etc
Good scope of coverage
2-3 aspects of globalization discussed
o Max 11 if only 2 aspects of globalization discussed
Ability to link to macroeconomic and microeconomic effects
Good depth of analysis
Good application to emerging economies context.

Evaluation
E1 Unexplained Judgement that is not supported by economic 1-2
analysis
E2 Evaluative assessment and judgement supported by economic 3-4
analysis. Excellent synthesis in which the student is able to
justify and arrive at a convincing stand (based on earlier
arguments and counter arguments)

Examiners comments:
Candidates were clearly more prepared for part (b) than (a) with a clear majority scoring in the high end of
the L2 range at least. Nonetheless, there were a number of common conceptual errors.
Some candidates provided rehearsed answers to a question on With globalization, to what extent do
developed countries have more to gain than developing countries? Every question is worded differently and
thus the approach cannot be identical. Do not go into the examination venue with the expectation that
questions will be repeated and reproducing rehearsed answers could guarantee a good grade.

The other major problem is that some candidates are simply stating certain outcomes instead of explaining
how to arrive at these outcomes using relevant economics analysis. Stating learnt content exhibits the lowest
skill ability. Higher skills of analysis, application and illustration need to be seen before a good grade can be
obtained. In the context of this question, many points were stated and not explained e.g. the exploitation of
labour, brain drain, and environmental degradation. What is required is an analysis of why such problems are
more prevalent in emerging economies and in the case of brain drain, a discussion on some of the effects on
the real economy in the short and long-run. Using the theoretical framework, classifying these points to
explain how AD or AS is impacted and the outcome on the various macro variables will increase the depth of
analysis and the economics rigour.

With regards to the issue of vulnerability to external shocks, the main problem for emerging economies is
that high dependence on trade which causes external events to have larger impact on the economy. Most
Year 6 Preliminary_9732 [Turn over]
RI 2012
40

candidates did not grasp this and simply explained how recessions in foreign countries would cause
recessions in the emerging economies. That explanation does not highlight the idea that the effects are
magnified due to the higher dependence on the tradeable sector and foreign firms.

A number of students also raised issues of protectionism. That is of limited relevance to a question on the
effects of globalization, which protectionism is clearly anathema to. Candidates must also note that
explaining the benefits of capital inflow and the costs of capital outflow (or any flow for that matter) does not
constitute balance. They are simply both sides of the same coin. Candidates should apply the concepts to
the context of emerging economies and explain the benefits and costs of the net flows.

The majority of candidates also failed to readdress the question in their conclusions. Instead of making a
reasoned judgment on whether emerging economies have more to lose from globalization, many students
suggested policies that emerging economies governments could adopt. Students must always keep in mind
that it is paramount to answer the given question.

Year 6 Preliminary_9732 [Turn over]


RI 2012

You might also like